Intégration

Exercices du dossier Intégration

Exercice 1329

9 novembre 2022 12:19 — Par Patrice Lassère

Bibliographie



[ID: 2531] [Date de publication: 9 novembre 2022 12:19] [Catégorie(s): Géométrie Topologie Continuité Dérivabilité Intégration Suites et séries Suites et séries de fonctions, séries entières Fonctions holomorphes Calcul différentiel Equations différentielles Analyse fonctionnelle Combinatoires et probabilités En cours... ] [ Nombre commentaires: 0] [nombre d'éditeurs: 1 ] [Editeur(s): Emmanuel Vieillard-Baron ] [nombre d'auteurs: 1 ] [Auteur(s): Patrice Lassère ]
Accordéon
Titre
Solution
Texte

Irrationalité de \(e\) (1) *

9 novembre 2022 16:17 — Par Patrice Lassère

Pour \(n\in\mathbb N\) on pose \(\displaystyle\ I_n=\int_0^\infty\,x^ne^xdx\).

Montrer que pour tout \(n\in\mathbb N\) il existe \(a_n, b_n\in\mathbb Z\) tels que \(I_n=a_n+eb_n\).

On suppose qu’il existe \(p,q\in\mathbb N^\star\) tels que \(e=p/q\), montrer que \[I_n\geq q^{-1},\ \forall\,n\in\mathbb N.\]

En déduire que \(e\not\in\mathbb Q\).



[ID: 2777] [Date de publication: 9 novembre 2022 16:17] [Catégorie(s): Intégration ] [ Nombre commentaires: 0] [nombre d'éditeurs: 1 ] [Editeur(s): Emmanuel Vieillard-Baron ] [nombre d'auteurs: 1 ] [Auteur(s): Patrice Lassère ]
Accordéon
Titre
Solution
Texte

Irrationalité de \(e\) (1)
Par Patrice Lassère le 9 novembre 2022 16:17

On procède par récurrence sur \(n\in\mathbb N\). Si \(n=0\), \(I_0=\int_0^1\,e^{x}dx=e-1\) et l’assertion est donc vraie pour \(n=0\) avec \(a_0=-1=-b_0\). Supposons la propriété vrai au rang \(n\) avec \(I_n=a_n+eb_n\), alors \[\begin{aligned}I_{n+1}&=\int_0^1\,e^xx^{n+1}dx=\left[ e^xx^{n+1}\right]_0^1-\int_0^1\,e^x(n+1)x^ndx\\ &=e-(n+1)I_n=e-(n+1)(a_n+eb_n)=-(n+1)a_n+e(1-(n+1)b_n) \end{aligned}\] soit la propriété au rang \(n+1\), C.Q.F.D.

Supposons que \(e=p/q\) avec \(p,q\in\mathbb N^\star\), alors \[0<I_n=a_n+eb_n=\dfrac{qa_n+pb_n}{q},\] qui implique \[qa_n+pb_n\geq 1\] et par conséquent \[I_n\geq \dfrac{1}{q},\quad\forall\,n\in\mathbb N.{(1)}\] D’un autre coté, on a la majoration immédiate \[0<I_n\leq \int_0^1\,ex^ndx=\dfrac{e}{n+1},\quad\forall\,n\in\mathbb N.{(2)}\] (1) et (2) donnent \[0<\dfrac{1}{q}\leq \dfrac{e}{n+1},\quad\forall\,n\in\mathbb N\] qui est visiblement absurde : \(e\) est donc irrationel.

Voir aussi pages [exp2] et [exp3] pour d’autres démontrations de l’irrationalité de \(e\).


Accordéon
Titre
Solution
Texte

Calcul de l’intégrale de Gauss \(\int_0^\infty e^{-t^2}dt\) (1)
Par Patrice Lassère le 9 novembre 2022 16:17

Notons \[f(x):=\int_0^\infty\dfrac{e^{-xt^2}}{1+t^2}dt=\int_0^\infty g(x,t)dt\]\(g(x,t)=\frac{e^{-xt^2}}{1+t^2}\) ; c’est une fonction \(\mathscr C^\infty\) sur \(\mathbb R_+\times\mathbb R_+\) qui vérifie \[\forall\,x\in\mathbb R_+,\qquad\vert g(x,t)\vert\leq \dfrac{1}{1+t^2}\in L^1(\mathbb R_+).\] \(f\) est donc (par convergence dominée) continue sur \(\mathbb R_+\). De même pour la dérivabilité, nous avons pour \(a>0\) \[\forall\,x\geq a>0\quad:\quad\left\vert\dfrac{\partial g}{\partial x}(x,t)\right\vert\leq\left\vert-\dfrac{t^2e^{-xt^2}}{1+t^2}\right\vert\leq \left\vert\dfrac{t^2e^{-at^2}}{1+t^2}\right\vert.\] Toujours par convergence dominée, \(g\) est de classe \(\mathscr C^1\) sur \([a,+\infty[\) et ceci pour tout \(a>0\) : \(f\in\mathscr C^1(\mathbb R_+^\star)\). En résumé \[f\in\mathscr C^0(\mathbb R_+)\cap\ \mathscr C^1(\mathbb R_+^\star)\quad\text{et}\quad \forall\,x\in\mathbb R_+^\star\ :\quad f'(x)=-\int_0^\infty\dfrac{t^2e^{-xt^2}}{1+t^2}dt.{(\text{\ding{56}})}\]

Notons \(I=\int_{\mathbb R_+}e^{-t^2}dt\). Avec () nous avons pour tout \(x>0\) \[f'(x)=-\int_0^\infty\dfrac{t^2e^{-xt^2}}{1+t^2}dt=f(x)-\int_0^\infty e^{-xt^2}dt \underset{u=t\sqrt{x}}{=}f(x)-\dfrac{I}{\sqrt{x}}.\] L’application \(g(x)=e^{-x}f(x)\) vérifie \[g'(x)=e^{-x}(f'(x)-f(x))=-e^{-x}\dfrac{I}{\sqrt{x}}.\] Il existe donc une constante \(C>0\) telle que \[\forall x\in\mathbb R_+^\star\ :\quad g(x)=C-I\int_0^x\dfrac{e^{-t}}{\sqrt{t}}dt.\] En outre \[\left( \vert g(x)\vert \leq e^{-x}\vert f(x)\vert\leq \dfrac{\pi}{2}e^{-x}\right) \quad\implies\quad\left( \lim_{x\to\infty}g(x)=0\right) {(1)}\] et \[g(x)=C-I\int_0^x\dfrac{e^{-t}}{\sqrt{t}}dt\underset{u=\sqrt{t}}{\ =\ }C-2I\int_0^{\sqrt{x}}e^{-u^2}du \underset{x\to\infty}{\longrightarrow}C-2I^2{(2)}.\] \((1)\) et \((2)\) donnent \(\ \displaystyle C=2I^2\). On a donc \[f(x)=e^x g(x)=2Ie^x\left(I-\int_0^{\sqrt{x}}e^{-u^2}du \right)=2Ie^{x}\int_{\sqrt{x}}^\infty,\quad\forall\,x>0,\] qui implique \[\lim_{x\to 0}f(x)=2I^2\] et par continuité de \(f\) à l’origine \[\lim_{x\to 0}f(x)=f(0)=\int_0^\infty\dfrac{dt}{1+t^2}=\dfrac{\pi}{2},\] soit \(\displaystyle I=\dfrac{\sqrt{\pi}}{2}\).


Accordéon
Titre
Solution
Texte

Calcul de l’intégrale de Cauchy \(\int_0^\infty \frac{sin(t)}{t}dt\) (1)
Par Patrice Lassère le 9 novembre 2022 16:17

Écrivons \(f(x)=\int_0^\infty g(x,t)dt\)\(g(x,t)=e^{-xt}\frac{sin(t)}{t}\). Pour \(x=0\), \(f(0)=\int_0^\infty\frac{\sin(t)}{t}dt\) et nous retrouvons l’intégrale (convergente1) de Cauchy ; pour \(x>0\), comme \(\vert g(x,t)\vert\leq e^{-xt}\in L^1(\mathbb R_+)\), \(f\) est encore bien définie : \(f\) est finalement définie sur \(\mathbb R_+\).

Soit \(a>0\), nous avons \[\vert g(x,t)\vert\leq e^{-at}\in L^1(\mathbb R_+)\quad\text{et}\quad\left\vert\dfrac{\partial g}{\partial x}(x,t)\right\vert=\vert -\sin(t)e^{-xt}\vert\leq e^{-at}\in L^1(\mathbb R_+).\] De ces deux inégalités, le théorème de continuité et dérivabilité des intégrales à paramètres assure que \(f\in\mathscr C^1(\mathbb R_+^\star)\) et \[\forall\,x\in\mathbb R_+^\star,\qquad f'(x)=-\int_0^\infty \sin(t)e^{-xt}dt.\] Remarque : Il faut se garder, malgré les questions suivantes, de vouloir par ces théorèmes de domination obtenir la continuité de \(f\) à l’origine : en effet \(f\) est à l’origine définie par l’intégrale de Cauchy qui est notoirement non absolument convergente et une domination de \(g\) dans un voisinage de l’origine impliquerai assurément l’absolue convergence. C’est pourquoi d’ailleurs les dominations n’ont lieu que sur \([a,+\infty[\)...

L’expression de \(f'(x)\) que nous venons d’obtenir nous permet un calcul explicite : soit \(x>0\) \[\begin{aligned}f'(x)= -\int_0^\infty \sin(t)e^{-xt}dt&=-\dfrac{1}{2i}\int_0^\infty \left( e^{it}-e^{-it}\right) e^{-xt}dt\\ &= -\dfrac{1}{2i}\int_0^\infty\left( e^{t(i-x)}-e^{-t(i+x)}\right) dt\\ &=-\dfrac{1}{2i}\left( \left[ \dfrac{e^{t(i-x)}}{i-x}\right]_0^\infty+\left[ \dfrac{e^{-t(i+x)}}{i+x}\right]_0^\infty \right) \\ &=-\dfrac{1}{2i}\left( -\dfrac{1}{i-x}-\dfrac{1}{i+x}\right) =-\dfrac{1}{1+x^2} \end{aligned}\] (les deux termes entre crochets sont nuls à l’infini car par exemple \(\vert\frac{e^{-t(i+x)}}{i+x}\vert=\frac{e^{-xt}}{\sqrt{x^2+1}}\to 0\) lorsque \(t\) tends vers \(+\infty\)....). En intégrant cette formule, il vient \[\exists\,C\in\mathbb R\ :\ \forall\,x\in\mathbb R_+^\star\qquad f(x)=-\text{arctan}(x)+C.\] La constante \(C\) n’est pas difficile à déterminer, en effet la formule ci-dessus implique que \[\lim_{x\to+\infty}f(x)=-\dfrac{\pi}{2}+C\] et pour tout \(x>0\) \[\vert f(x)\vert\leq\int_0^\infty e^{-xt}dt=\dfrac{1}{x}\underset{x\to +\infty}{\to}0\] soit \[-\dfrac{\pi}{2}+C=0\qquad\text{et}\quad C=\dfrac{\pi}{2}.\] Résumons nous : \[f(x)=\begin{cases}-\text{arctan}(x)+\dfrac{\pi}{2},\quad &\text{si}\ x>0\\ \int_0^\infty\dfrac{\sin(t)}{t}dt,\quad &\text{si}\ x=0.\end{cases}{(\text{\ding{56}})}\]

Il s’agit de montrer que \[\lim_{x\to 0_+}\vert f(x)-f(0)\vert=\left\vert\int_0^\infty\dfrac{\sin(t)}{t}\left( e^{-xt}-1\right)\right\vert dt=0.\] Cette limite n’est pas triviale, on va faire une intégration par parties : considérons pour \(t>0\), \(G(t)=\int_t^\infty\frac{\sin(u)}{u}du\). \(G\) est dérivable et \(G'(t)=-\frac{\sin(t)}{t}\), en outre la convergence de \(\int_0^\infty\frac{\sin(t)}{t}dt\) implique \(\lim_{t\to\infty}G(t)=0\). Ainsi \[\begin{aligned} f(x)-f(0)&=\int_0^\infty\dfrac{\sin(t)}{t}\left( e^{-xt}-1\right)\\ &=-\int_0^\infty G'(t)\left( e^{-xt}-1\right)\\ &=\left[ G(t)\left( e^{-xt}-1\right)\right]_0^\infty-\int_0^\infty G(t)xe^{-xt}dt\\ &\underset{u=xt}{=}-\int_0^\infty G\left( \dfrac{u}{x}\right) e^{-u}du:=-\int_0^\infty H(x,u)du \end{aligned}\] et la fonction \[H(x,u)=\begin{cases} G\left( \dfrac{u}{x}\right)e^{-u}\quad & \text{si }\ x\neq 0,\\ 0\quad&\text{si }\ x= 0.\end{cases}\] est continue sur \(\mathbb R^+\times\mathbb R_+^\star\) (la continuité en \((0,u)\) découle de \(\lim_{t\to\infty}G(t)=0\)) ; elle est aussi dominée par \[\vert H(x,u)\vert\leq e^{-u}\in L^1(\mathbb R_+).\] Donc par convergence dominée \[\lim_{x\to 0_+}\vert f(x)-f(0)\vert=\lim_{x\to 0}\left\vert-\int_0^\infty H(x,u)du \right\vert=\left\vert-\int_0^\infty \lim_{x\to 0}H(x,u)du \right\vert=0.{(\text{\ding{52}})}\] \(f\) est donc bien continue à l’origine.

() et () donnent immédiatement \[\int_0^\infty\dfrac{\sin(t)}{t}dt=\dfrac{\pi}{2}.\]


  1. 1  Voir l’exercice ????

Encore un calcul de l’intégrale de Cauchy \(\int_0^{+\infty}\frac{sin(t)}{t}dt\) *

9 novembre 2022 16:17 — Par Patrice Lassère

Montrer que \[\int_0^{+\infty}\frac{sin(t)}{t}dt=\dfrac{\pi}{2}.\]



[ID: 2783] [Date de publication: 9 novembre 2022 16:17] [Catégorie(s): Intégration ] [ Nombre commentaires: 0] [nombre d'éditeurs: 1 ] [Editeur(s): Emmanuel Vieillard-Baron ] [nombre d'auteurs: 1 ] [Auteur(s): Patrice Lassère ]
Accordéon
Titre
Solution
Texte

Encore un calcul de l’intégrale de Cauchy \(\int_0^{+\infty}\frac{sin(t)}{t}dt\)
Par Patrice Lassère le 9 novembre 2022 16:17

La semi-convergence de cette intégrale est classique. Remarquons que \[\int_0^{\infty}\frac{\sin(t)}{t}dt=\lim_{n\to\infty}\int_0^{n\pi/2}\frac{\sin(t)}{t}dt =\lim_{n\to\infty}\int_0^{\pi/2}\frac{\sin(nx)}{x}dx.\] Nous allons successivement montrer \[\int_0^{\pi/2}\frac{\sin((2n+1)x)}{\sin(x)}dx =\dfrac{\pi}{2},\quad\forall\,n\in\mathbb N,{\text{(\ding{52})}}\] puis \[\lim_{n\to\infty}\left( \int_0^{\pi/2}\frac{\sin((2n+1)x)}{x}dx- \int_0^{\pi/2}\frac{\sin((2n+1)x)}{\sin(x)}dx\right) =0,{\text{(\ding{56})}}\] ce qui fournira la formule désirée.

Pour (), puisque pour tout \(x\in]0,\pi/2[\) \[\dfrac{1}{2}+\cos(x)+\cos(2x)+\dots+\cos(nx)= \dfrac{\sin(2^{-1}(2n+1)x)}{2\sin(x/2)},\] soit \[\dfrac{\sin(2n+1)x}{\sin(x)}=1+2\cos(2x)+2\cos(4x)+\dots+2\cos(2nx),\quad x\in]0,\pi[\] qui donne immédiatement ().

Pour la seconde on peut écrire \[\begin{aligned}\lim_{n\to\infty}\left( \int_0^{\pi/2}\frac{\sin((2n+1)x)}{x}dx- \int_0^{\pi/2}\frac{\sin((2n+1)x)}{\sin(x)}dx\right)\\ &=\lim_{n\to\infty}\int_0^{\pi/2} \dfrac{\sin(x)-x}{x\sin(x)}\sin((2n+1)x)dx\end{aligned}\] et cette dernière limite est nulle par le lemme de Riemann-Lebesgue ([waint], page ???). D’où ().


Toujours un calcul de l’intégrale de Cauchy\(\int_0^{+\infty}\frac{sin(t)}{t}dt\) (3) *

9 novembre 2022 16:17 — Par Patrice Lassère

Sachant que pour tout \(a>0\)

\[\int_0^\infty {{\sin(t)}\over{t+a}}dt=\int_0^\infty{{e^{-at}}\over t^2+1}dt,\]

montrer que

\[\int_0^\infty {{\sin(t)}\over{t}}dt={\pi\over 2}.\]



[ID: 2785] [Date de publication: 9 novembre 2022 16:17] [Catégorie(s): Intégration ] [ Nombre commentaires: 0] [nombre d'éditeurs: 1 ] [Editeur(s): Emmanuel Vieillard-Baron ] [nombre d'auteurs: 1 ] [Auteur(s): Patrice Lassère ]
Accordéon
Titre
Solution
Texte

Toujours un calcul de l’intégrale de Cauchy\(\int_0^{+\infty}\frac{sin(t)}{t}dt\) (3)
Par Patrice Lassère le 9 novembre 2022 16:17

On à immédiatement pour \(a>0\)

\[\lim_{a\to 0_+} {{e^{-at}}\over t^2+1}={1\over t^2+1}=f(t),\quad \forall\,t\in\mathbb R_+,\quad\text{ et }\quad\left\vert{{e^{-at}}\over t^2+1}\right\vert\leq f(t)\in L^1(\mathbb R_+)\]

donc, par convergence dominée

\[\lim_{a\to 0_+} \int_0^\infty{{e^{-at}}\over t^2+1}=\int_0^\infty{1\over t^2+1}={\pi\over 2}.\]

Pour le terme de gauche, une convergence dominée est sans espoir car il est notoire (cf. exercice page [ExoIntegralesImpropresDontSinusTsurT]) que \[t\mapsto{{\sin(t)}\over t}\not\in L^1(\mathbb R_+).\] On peut tout de même justifier l’inversion des deux limites à la main par exemple en remarquant que

\[\left\vert\int_0^\infty{\sin(t)\over t+a}dt-\int_0^\infty{\sin(t)\over t}dt\right\vert\leq \int_0^1{a\vert\sin(t)\vert\over t(t+a)}dt +\int_1^\infty{a\vert\sin(t)\vert\over t(t+a)}dt\]

le premier terme du second membre tend vers zéro avec \(a\) par convergence dominée car l’intégrande converge simplement vers la fonction nulle sur \([0,1]\) avec la domination \[{a\vert\sin(t)\vert\over t(t+a)}\leq {\vert\sin(t)\vert\over t}\in L^1([0,1]).\]

Pour le second terme, l’affaire est encore plus simple puisque \[\int_1^\infty{a\vert\sin(t)\vert\over t(t+a)}dt \leq a\int_0^\infty {dt\over t^2}\rightarrow_{a\to 0} 0.\]

On peut aussi, sans convergence dominée écrire pour tout \(\varepsilon>0\)

\[\begin{aligned}\left\vert\int_0^\infty{\sin(t)\over t+a}dt-\int_0^\infty{\sin(t)\over t}dt\right\vert &\leq \int_0^\varepsilon{a\vert\sin(t)\vert\over t(t+a)}dt +\int_\varepsilon^\infty{a\vert\sin(t)\vert\over t(t+a)}dt\\ &\leq \int_0^\varepsilon{\sin(t)\over t}dt+\int_\varepsilon^\infty{a\over t^2}dt \\ &\leq \varepsilon+{a\over\varepsilon}\quad \quad\forall\ a,\varepsilon>0\\ &\leq 2\sqrt{a}\qquad \quad\forall\ a>0\ (\text{on a fait }\varepsilon=\sqrt{a})\end{aligned}\]

d’où le résultat.

Voir l’exercice suivant pour démontrer :

\[\int_0^\infty {{\sin(t)}\over{t+a}}dt=\int_0^\infty{{e^{-at}}\over t^2+1}dt, \forall\,a\in\mathbb R_+^\star.\]


Accordéon
Titre
Solution
Texte

Calcul de l’intégrale de Cauchy \(\int_0^\infty\,\frac{\sin(t)}{t}dt\) (4)
Par Patrice Lassère le 9 novembre 2022 16:17

Ces intégrales impropres sont clairement convergentes pour tout \(x\in\mathbb R_+\) ; posons pour \((x,t)\in\mathbb R_+\times\mathbb R_+\) : \(f(x,t)=\sin(xt)/t+x, g(x,t)=e^{-tx}/t^2+1\). Les dominations \[\begin{aligned}&\left\vert g(x,t)\right\vert\leq \dfrac{1}{1+t^2},\quad\forall\,t\in\mathbb R_+,\\ &\quad\forall\,t\in\mathbb R_+,\quad \left\vert \dfrac{\partial g(x,t)}{\partial x}\right\vert\leq \dfrac{te^{-at}}{1+t^2}\in L^1(\mathbb R_+),\quad\forall\,x\geq a>0,\\ &\quad\forall\,t\in\mathbb R_+,\quad\left\vert \dfrac{\partial^2 g(x,t)}{\partial x^2}\right\vert\leq \dfrac{t^2e^{-at}}{1+t^2}\in L^1(\mathbb R_+),\quad\forall\,x\geq a>0, \end{aligned}\] assurent par convergence dominée que \(g\) est continue sur \(\mathbb R_+\) et de classe \(\mathscr C^2\) sur \(\mathbb R_+^\star\) avec \[g'(x)=-\int_0^\infty\,\dfrac{te^{-xt}}{t^2+1}dt,\ g''(x)=\int_0^\infty\,\dfrac{t^2e^{-xt}}{t^2+1}dt,\quad\forall\,x\in\mathbb R_+^\star.\] On en déduit immédiatement que \(g''(x) +g(x)=1/x\) sur \(\mathbb R_+^\star\).

Pour \(f\) c’est un peu plus délicat car l’application \(t\mapsto f(x,t)\) est notoirement non absolument intégrable sur \(\mathbb R_+\) et toute domination est veine, on commence donc par une intégration par parties pour obtenir une expression plus exploitable de \(f\). \[f(x)=\int_0^\infty\,\dfrac{\sin(t)}{t+x}dt=\left[ \dfrac{1-\cos(t)}{t+x}\right]_0^{\infty} +\int_0^\infty\,\dfrac{1-\cos(t)}{(t+x)^2}dt=\int_0^\infty\,\dfrac{1-\cos(t)}{(t+x)^2}dt\] (afin d’alléger les calculs on a choisi \(1-\cos(t)\) comme primitive de \(\sin(t)\) choix qui annule le terme entre crochets). De là, si \(h(x,t)={1-\cos(t)}/{(t+x)^2}\) et pour \(x\geq a>0\) \[\begin{aligned} &\forall\,t\in\mathbb R_+,\quad\left\vert \dfrac{\partial h(x,t)}{\partial x}\right\vert =\left\vert-\dfrac{2(1-\cos(t))}{(t+x)^3}\right\vert\leq \dfrac{2}{(t+a)^3}\in L^1(\mathbb R_+),\quad\forall\,x\geq a>0,\\ &\forall\,t\in\mathbb R_+,\quad\left\vert \dfrac{\partial^2 h(x,t)}{\partial x^2}\right\vert =\left\vert\dfrac{6(1-\cos(t))}{(t+x)^4}\right\vert\leq \dfrac{12}{(t+a)^3}\in L^1(\mathbb R_+),\quad\forall\,x\geq a>0, \end{aligned}\] ces dominations impliquent que \(f\in\mathscr C^2(\mathbb R_+^\star)\) avec \[f''(x)=\int_0^\infty\, \dfrac{6(1-\cos(t))}{(t+x)^4}dt,\quad\forall\,x\in\mathbb R_+^\star\] et avec une intégration par parties \[\begin{aligned} f''(x)&=\int_0^\infty\, \dfrac{6(1-\cos(t))}{(t+x)^4}dt=\left[ -\dfrac{2(1-\cos(t))}{(t+x)^3}\right]_0^\infty+\int_0^\infty\,\dfrac{2\sin(t)}{(t+x)^3}dt\\ &=\left[ -\dfrac{\sin(t)}{(t+x)^2}\right]_0^\infty+ \int_0^\infty\,\dfrac{\cos(t)}{(t+x)^2}dt=\int_0^\infty\,\dfrac{\cos(t)}{(t+x)^2}dt\\ &=\int_0^\infty\,\dfrac{dt}{(t+x)^2}-\int_0^\infty\,\dfrac{1-\cos(t)}{(t+x)^2}dt\\ &=\dfrac{1}{x}-f(x),\quad x>0. \end{aligned}\]

\(f\) et \(g\) sont solutions sur \(\mathbb R_+^\star\) de l’équation \(y''+y=1/x\), \(f-g\) est donc solution de l’équation \(y''+y=0\) : c’est la restriction à \(\mathbb R_+^\star\) d’une solution sur \(\mathbb R\) de \(y''+y=0\) donc \(2\pi\)-périodique.

Soit \(x>0\), vu ce qui précède \[f(x)=\dfrac{1}{x}-\int_0^\infty\,\dfrac{2\sin(t)}{(t+x)^3}dt\] et comme \[\left\vert\int_0^\infty\,\dfrac{2\sin(t)}{(t+x)^3}dt\right\vert\leq \int_0^\infty\,\dfrac{2dt}{(t+x)^3}=\dfrac{2}{x^2}=o(x^{-1})\] i.e. \[f(x)=\dfrac{1}{x}+o(x^{-1})\underset{+\infty}{\sim}\dfrac{1}{x}.\] Pour \(g\), on procède de même encore plus simplement.

Sur \(\mathbb R_+^\star\), \(f-g\) est continue \(2\pi\)-périodique et tends vers \(0\) en \(+\infty\) : elle est donc identiquement nulle et on a \[\int_0^\infty\,\dfrac{\sin(t)}{t+x}dt=\int_0^\infty\,\dfrac{e^{-xt}}{t^2+1}dt,\quad\forall\,x\in\mathbb R_+^\star.\]

Pour conclure, voir l’exercice précédent.


Accordéon
Titre
Solution
Texte

Calcul de l’intégrale de Cauchy \(\int_0^\infty\,\frac{\sin(t)}{t}dt\) (5)
Par Patrice Lassère le 9 novembre 2022 16:17

L’intégrale définissant \(F\) est clairement convergente pour tout \(x\in\mathbb R\) : \(F\) est définie sur \(\mathbb R\) et est impaire. Posons \(f(x,t)=\frac{\sin(xt)}{t(t^2+1)}\).

Soit \(a>0\), pour \(x\in [-a,a]\) et \(t\in\mathbb R_+\) on a \[\left\vert f(x,t)\right\vert =\left\vert \dfrac{\sin(xt)}{t}\cdot\dfrac{1}{t^2+1}\right\vert \leq \dfrac{\vert x\vert}{t^2+1}\leq \dfrac{a}{t^2+1}\in L^1(\mathbb R_+),\] vu la régularité de \(f\) le théorème de continuité des intégrales à paramètres assure que \(F\in\mathscr C^0([-a,a])\), et ceci pour tout \(a>0\) : \(F\) est donc continue sur \(\mathbb R\).

\(\partial_x f(x,t)=\dfrac{\cos(xt)}{t^2+1}\), on a donc pour tout \(x\in\mathbb R\) et \(t\in\mathbb R_+\) \[\left\vert \partial_x f(x,t)\right\vert=\left\vert\dfrac{\cos(xt)}{t^2+1}\right\vert\leq \dfrac{1}{t^2+1}\in L^1(\mathbb R),\] par le théorème de dérivation des intégrales à paramètres, \(F\in\mathscr C^1(\mathbb R)\) et \(\displaystyle F'(x)=\int_0^\infty\,\dfrac{\cos(xt)}{t^2+1}dt.\)

Pour l’existence de la dérivée seconde l’affaire est plus délicate, car \[\left\vert \partial^2_x f(x,t)\right\vert=\left\vert\dfrac{-t\sin(xt)}{t^2+1}\right\vert\underset{t\to\infty}{\sim}\left\vert \dfrac{\sin(xt)}{t}\right\vert,\] et cette dernière n’est (comme \(\left\vert\frac{\sin(t)}{t}\right\vert\)) pas intégrable en \(+\infty\) : toute tentative de domination (même locale) pour appliquer le théorème précédent est donc vaine. L’astuce constiste par une intégration par parties à écrire \(F'\) sous une forme acceptable pour justifier la dérivation sous l’intégrale :soit \(x\in\mathbb R^\star\) : \[\begin{aligned}F'(x)=\int_0^\infty\,\dfrac{\cos(xt)}{t^2+1}dt &= \left[ \dfrac{\sin(xt)}{x(t^2+1)}\right]_0^\infty+\int_0^\infty\,\dfrac{2t\sin(xt)}{x(t^2+1)^2}dt\\ &=\int_0^\infty\,\dfrac{2t\sin(xt)}{x(t^2+1)^2}dt. \end{aligned}\] Ainsi, pour \(x\not= 0\) on a \[F'(x)=\int_0^\infty\,\dfrac{\cos(xt)}{t^2+1}dt= \int_0^\infty\,\dfrac{2t\sin(xt)}{x(t^2+1)^2}dt{(\text{\ding{56}})}\] sous cette seconde forme, on va pouvoir appliquer le théorème de dérivation des intégrales à paramètres, en effet soit \(a>0\), pour \(x\geq a\) \[\begin{aligned}\left\vert \dfrac{\partial}{\partial x}\left( \dfrac{2t}{x}\cdot \dfrac{\sin(xt)}{(t^2+1)^2}\right) \right\vert &\leq \left\vert \dfrac{2t}{x^2}\cdot \dfrac{\sin(xt)}{(t^2+1)^2}\right\vert+\left\vert \dfrac{2t^2\cos(xt)}{x(t^2+1)^2}\right\vert\\ &\leq \dfrac{\vert 2t\vert }{a^2(t^2+1)^2}+\dfrac{2t^2}{a(t^2+1)^2}\in L^1(\mathbb R) \end{aligned}\] on peut donc dériver sous l’intégrale : \(F\) est deux fois dérivable sur \(\mathbb R^\star\) et \[F''(x)=\int_0^\infty\, \left(-\dfrac{2t}{x^2}\cdot \dfrac{\sin(xt)}{(t^2+1)^2}+ \dfrac{2t^2\cos(xt)}{x(t^2+1)^2}\right)dt,\quad\forall\,x\in\mathbb R^\star.\] Cette expression est un peu chargée, faisons une intégration par parties : \[\begin{aligned}F''(x)&=\int_0^\infty\, \left(-\dfrac{2t}{(t^2+1)^2}\cdot \dfrac{\sin(xt)}{x^2}+ \dfrac{2t^2\cos(xt)}{x(t^2+1)^2}\right)dt\\ &=\left[ \dfrac{\sin(xt)}{x^2(t^2+1)}-\dfrac{t\cos(xt)}{x(t^2+1)} \right]_0^\infty -\int_0^\infty\,\left( \dfrac{x\cos(xt)}{x^2(t^2+1)}-\dfrac{\cos(xt)-xt\sin(xt)}{x(t^2+1)} \right)dt\\ &=-\int_0^\infty\,\dfrac{t\sin(xt)}{t^2+1}dt. \end{aligned}\] Il est intéressant à ce stade d’observer que nous retrouvons finalement la formule \[F''(x)=\int_0^\infty\dfrac{\partial^2}{\partial x^2}f(x,t)dt,\quad x\in\mathbb R^\star,\] mais pour justifier une dérivation sous l’intégrale une transformation de \(F'\) (voir ()) à été nécessaire ; remarquez aussi que l’existence de \(F''(0)\) reste ouverte. Nous avons donc : \[\begin{aligned} &F'(x)=\int_0^\infty\,\dfrac{\cos(xt)}{t^2+1}dt,\quad\forall\,x\in\mathbb R,\\ &F''(x)=-\int_0^\infty\,\dfrac{t\sin(xt)}{t^2+1}dt,\quad\forall\,x\in\mathbb R^\star. \end{aligned}\]

Ainsi, pour tout \(x\in\mathbb R^\star\), \[F(x)-F''(x)= \int_0^\infty\,\dfrac{\sin(xt)}{t(t^2+1)}dt+\int_0^\infty\,\dfrac{t\sin(xt)}{x(t^2+1)}dt=\int_0^\infty\,\dfrac{\sin(xt)}{t}dt=\begin{cases}C,\quad\forall\,x\in\mathbb R_+^\star,\\ -C,\quad\forall\,x\in\mathbb R_-^\star.\end{cases}\] \(F\) est donc solution de l’équation différentielle \(F-F''=C\) sur \(\mathbb R_+^\star\) et \(F-F''=-C\) sur \(\mathbb R_-^\star\) ce qui nous donne \[F(x)=\begin{cases} ae^x+be^{-x}+C,\quad\forall\,x\in\mathbb R_+^\star,\\ ce^x+de^{-x}-C,\quad\forall\,x\in\mathbb R_-^\star. \end{cases}\] (remarquez que ces équations impliquent \(\lim_{0_+}F''(x)=C=-\lim_{0_-}F''(x)\) qui assurent si \(C\neq 0\) que \(F''\) admet à l’origine des limites à droite et à gauche différentes ce qui (propriété classique de l’application dérivéée, Darboux par exemple) nous permet d’affirmer que \(F''(0)\) n’existe pas mais \(F'\) est tout de même dérivable à droite et à gauche en \(0\) avec \(F''(0_+)=C=-F''(0_-)\)...) \(F\) étant impaire, \(a=-d, b=-c\) soit \[F(x)=\begin{cases} ae^x+be^{-x}+C,\quad\forall\,x\in\mathbb R_+^\star,\\ -be^x-ae^{-x}-C,\quad\forall\,x\in\mathbb R_-^\star \end{cases}\] et \(F\) continue à l’origine avec \(F(0)=0\) implique \[F(0)=0=\lim_{x\to 0_+}F(x)=a+b+C=\lim_{x\to 0_-}F(x)=-a-b-C\] soit \(a+b=-C\) ; de même, \(F'\) continue à l’origine avec \(F'(0)=\pi/2\) donne \(a-b=\pi/2\) i.e. \(2a=\pi/2-C, 2b=-C-\pi/2\) et finalement \[F(x)=\dfrac{\pi}{2}\text{sh}(x)-C\text{ch}(x)+C,\quad\,\forall\,x\in\mathbb R_+^\star.{(\text{\ding{52}})}\]

Il reste à évaluer \(C\). Pour cela, montrons que \(\displaystyle\lim_{x\to+\infty}F(x)=C\). Soit \(x>0\), \[F(x)-C=F''(x)=\int_0^\infty\, \left(-\dfrac{2t}{x^2}\cdot \dfrac{\sin(xt)}{(t^2+1)^2}+ \dfrac{2t^2\cos(xt)}{x(t^2+1)^2}\right)dt\] (on a encore ici besoin de la première expression de \(F''\) pour conclure facilement) pour \(x\geq a>0\), on a la domination \[\left\vert -\dfrac{2t}{x^2}\cdot \dfrac{\sin(xt)}{(t^2+1)^2}+ \dfrac{2t^2\cos(xt)}{x(t^2+1)^2}\right\vert\leq \dfrac{2t}{a^2(t^2+1)^2}+\dfrac{2t^2}{a(t^2+1)^2}\in L^1(\mathbb R_+).\] Donc par le théorème de la convergence dominée \[\lim_{x\to+\infty}(F(x)-C)=\int_0^\infty\,\lim_{x\to +\infty}\left(-\dfrac{2t}{x^2}\cdot \dfrac{\sin(xt)}{(t^2+1)^2}+ \dfrac{2t^2\cos(xt)}{x(t^2+1)^2}\right)dt=0\] soit avec () \[\lim_{x\to+\infty}F(x)=C\quad \text{et}\quad F(x)\underset{+\infty}{\sim}\left( \dfrac{\pi}{2}-C\right) e^x+C\] qui donnent \[C=\int_0^\infty\,\dfrac{\sin(t)}{t}dt=\dfrac{\pi}{2}.\] C.Q.F.D.


Calcul de l’intégrale de Cauchy \(\int_0^\infty\,\frac{\sin(t)}{t}dt\) (5), *

9 novembre 2022 16:17 — Par Patrice Lassère

Montrer que pour tout \(x\in\mathbb R\) \[\int_0^{\pi/2}\,e^{-x\cos(t)}\cos(x\sin(t))dt=\dfrac{\pi}{2}-\int_0^x\,\dfrac{\sin(t)}{t}dt.\] En déduire que \(\displaystyle \int_0^\infty \frac{sin(t)}{t}dt=\frac{\pi}{2}\).



[ID: 2791] [Date de publication: 9 novembre 2022 16:17] [Catégorie(s): Intégration ] [ Nombre commentaires: 0] [nombre d'éditeurs: 1 ] [Editeur(s): Emmanuel Vieillard-Baron ] [nombre d'auteurs: 1 ] [Auteur(s): Patrice Lassère ]
Accordéon
Titre
Solution
Texte

Calcul de l’intégrale de Cauchy \(\int_0^\infty\,\frac{\sin(t)}{t}dt\) (5),
Par Patrice Lassère le 9 novembre 2022 16:17

Soit \(x\in\mathbb R\), on a \[\begin{aligned} \int_0^{\pi/2}\,e^{-x\cos(t)}\cos(x\sin(t))dt &=\text{Re}\left( \int_0^{\pi/2}\,e^{-x\cos(t)}e^{ix\sin(t)}dt\right)\\ &=\text{Re}\left( \int_0^{\pi/2}\,e^{-xe^{-it}}dt\right)\\ &=\text{Re}\left( \int_0^{\pi/2}\,\sum_{n=0}^\infty\dfrac{(-x)^ne^{-int}}{n!}dt\right)\\ &=\dfrac{\pi}{2}+\sum_{n=1}^\infty\dfrac{(-x)^n}{n!} \int_0^{\pi/2}\,\text{Re}\left(e^{-int}\right)dt\\ &=\dfrac{\pi}{2}+\sum_{n=1}^\infty\dfrac{(-x)^n}{n!} \left[ \dfrac{\sin(nt)}{n}\right]_0^{\pi/2}\\ &=\dfrac{\pi}{2}+\sum_{n=1}^\infty\dfrac{(-x)^n}{n!} \dfrac{\sin(n\pi/2)}{n}\\ &=\dfrac{\pi}{2}+\sum_{k=0}^\infty\dfrac{(-x)^{2k+1}}{(2k+1)!} \dfrac{\sin((2k+1)\pi/2)}{2k+1}\\ &=\dfrac{\pi}{2}-\sum_{k=0}^\infty\dfrac{(-1)^kx^{2k+1}}{(2k+1)(2k+1)!}\\ &=\dfrac{\pi}{2}-\sum_{k=0}^\infty\,\int_0^x\,\dfrac{(-1)^kt^{2k+1}}{(2k+1)!}\dfrac{dt}{t}\\ &=\dfrac{\pi}{2}-\int_0^x\,\sum_{k=0}^\infty\,\dfrac{(-1)^kt^{2k+1}}{(2k+1)!}\dfrac{dt}{t}\\ &=\dfrac{\pi}{2}-\int_0^x\,\dfrac{\sin(t)}{t}dt. \end{aligned}\] Les deux échanges \(\int\sum=\sum\int\) sont justifiés par la normale convergence des deux séries entières sur le domaine d’intégration (leur rayon de convergence étant infini).

Une convergence dominée élémentaire1 (\(\left\vert e^{-x\cos(t)}\cos(x\sin(t))\right\vert\leq 1\in L^1([0,\pi/2])\)) implique \[\lim_{x\to+\infty} \int_0^{\pi/2}\,e^{-x\cos(t)}\cos(x\sin(t))dt=0,\] soit, vu la formule établie au dessus \[\lim_{x\to+\infty}\left( \dfrac{\pi}{2}-\int_0^x\,\dfrac{\sin(t)}{t}dt\right)=0\] on retrouve bien la valeur de l’intégrale de Cauchy \(\displaystyle \int_0^\infty\,\dfrac{\sin(t)}{t}dt= \dfrac{\pi}{2}\).


  1. 1  que l’on peut aussi éviter en coupant l’intégrale en deux...

Étude de la suite \(( u_n={n\over2}-\sum_{k=1}^n{n^2\over(n+k)^2})_n\) *

9 novembre 2022 16:17 — Par Patrice Lassère

Convergence et limite de la suite de terme général

\[u_n={n\over2}-\sum_{k=1}^n{n^2\over(n+k)^2}.\]



[ID: 2793] [Date de publication: 9 novembre 2022 16:17] [Catégorie(s): Intégration ] [ Nombre commentaires: 0] [nombre d'éditeurs: 1 ] [Editeur(s): Emmanuel Vieillard-Baron ] [nombre d'auteurs: 1 ] [Auteur(s): Patrice Lassère ]
Accordéon
Titre
Solution
Texte

Étude de la suite \(( u_n={n\over2}-\sum_{k=1}^n{n^2\over(n+k)^2})_n\)
Par Patrice Lassère le 9 novembre 2022 16:17

Ecrivons \[u_n=n \left( {1\over2}-{1\over n}\sum_{k=1}^n{1\over\left(1+{k\over n}\right)^2} \right) =n \left( \int_0^1f(t)dt-{1\over n}\sum_{k=1}^nf({k\over n}) \right)\]\(f(t)={{(1+t)^{-2}}}.\) En posant pour \(0<x<1,\ F(x)=\int_0^x f(t)dt\), on peut écrire cette dernière expression sous la forme :

\[\begin{aligned} u_n= n\left(\int_0^1f(t)dt-{1\over n}\sum_{k=1}^nf({k\over n})\right)&=n\left(\sum_{k=0}^{n-1}\int_{k/n}^{k+1/n}f(t)dt-{1\over n}\sum_{k=1}^{n}F'({k\over n})dt\right)\\ &= f(0)-f(1)+n\left(\sum_{k=0}^{n-1}\int_{k/n}^{k+1/n}f(t)dt-{1\over n}\sum_{k=0}^{n-1}F'({k\over n})\right)\\ &=f(0)-f(1)+n\left(\sum_{k=0}^{n-1} F({k+1\over n})-F({k\over n})-{1\over n}F'({k\over n})\right)\end{aligned}\]

la formule de Taylor-Lagrange appliquée à \(F\) à l’ordre \(2\) nous assure que

\[F({k+1\over n})-F({k\over n})-{1\over n}F'({k\over n})={1\over 2n^2}F''(\zeta_{n,k}) \quad\text{ où }\quad\zeta_{n,k}\in]{k\over n},{k+1\over n}[\]

et finalement \[\lim_{n\to+\infty}u_n=f(0)-f(1)+{1\over 2}\lim_{n\to+\infty}{1\over n}\sum_{k=0}^{n-1}F''(\zeta_{n,k})=f(0)-f(1)+{1\over 2}\int_0^1F"(t)dt={3\over 8}\] la dernière limite étant justifiée puisque l’on y reconnaît la somme de Riemann de la fonction continue \(F''\) associée à la subdivision \(\{{k\over n}\}_{k=0}^{n}\) en les points \(\left(\zeta_{k,n}\in]{k\over n},{k+1\over n}[\right)_k\).


Accordéon
Titre
Solution
Texte

Autour du théorème des moments de Hausdorff
Par Patrice Lassère le 9 novembre 2022 16:17

Soit \(f\) une telle fonction, avec la linéarité de l’intégrale on a

\[\int_a^b f(t)P(t)dt=0,\quad\forall P\in\mathbb R[X],\]

mais par le théorème de Weierstrass, il existe une suite \((P_n)_n\subset\mathbb R[X]\) qui converge uniformément sur \([a,b]\) vers \(f\), et par convergence uniforme sur \([a,b]\) \[\int_a^b f^2(t)dt=\int_a^b \lim_n f(t)P_n(t)dt=\lim_n\int_a^bf(t)P_n(t)dt =0\]

soit (\(f^2\) est continue et positive) \(f\equiv 0\).

Pour tout \(n\in\mathbb N\ :\ \) \[\left\vert t^ne^{-\omega t}\right\vert= t^n\exp(-{t\sqrt 2\over 2})\in L^1(\mathbb R)\] et l’intégrale \(I_n\) est bien convergente ; en outre, si \(n\geq 1\) une intégration par parties donne

\[I_n=\omega^{-1}\left[-t^ne^{-\omega t}\right]_0^\infty+{n\over\omega}\int_0^\infty t^{n-1}e^{-\omega t}dt={n\over\omega}I_{n-1}\]

soit

\[I_n={n!\over {\omega^{n+1}}},\quad n\in\mathbb N.\]

Et puisque pour \(n\geq 1\ :\ \omega^{4(n+1)}=(-1)^{n+1}\) au vu du calcul précédent

\[I_{4n+3}\in\mathbb R,\ \forall\,n\in\mathbb N\]

sa partie imaginaire est donc toujours nulle i.e.

\[\begin{aligned}0=im(I_{4n+3})&=\int_0^\infty t^{4n+3} \exp(-{t\sqrt 2\over 2})\sin({t\sqrt 2\over 2})dt \\ &=\int_0^\infty x^n\sin(x^{1/4})\exp({-x^{1/4}})dx\quad(\text{poser }x=t^4/4)\\ &=\int_0^\infty x^nf(x)dx\quad n\in\mathbb N \end{aligned}\]

d’où le contrexemple désiré.

Commencons par remarquer que, vu les hypothèses sur \(f\) \[\begin{aligned} \int_{-a}^a\left( f(t)+f(-t)\right)t^{2n} dt&=\int_{-a}^a f(t)t^{2n}dt+\int_{-a}^a f(-t)(-t)^{2n}dt \\ &=2\int_{-a}^af(t)t^{2n}dt=0,\quad\forall\,n\in\mathbb N.\end{aligned}\] D’autre part \[\int_{-a}^a\left( f(t)+f(-t)\right)t^{2n+1} dt=\int_{-a}^a f(t)t^{2n+1}dt-\int_{-a}^a f(-t)(-t)^{2n+1}dt =0,\quad\forall\,n\in\mathbb N.\] L’application continue sur \([-a,a]\), \(g(t)=f(t)-f(-t)\) vérifie donc \[\int_{-a}^af(t)t^{n} dt=0,\qquad\forall\,n\in\mathbb N,\] elle est donc (vu la première question) identiquement nulle : \(f\) est bien impaire.

La procédure est identique pour le second cas.

 Pour la première question, on est pas obligé d’utiliser Weierstrass, on peut procéder de la manière suivante : soit \(f\in\mathscr C([0,1])\) une application vérifiant \(\int_0^1t^nf(t)dt=0,\quad\forall\,n\in\mathbb N\). Par linéarité de l’intégrale \(\int_0^1 P(t)f(t)dt=0,\quad\forall\,P\in\mathbb R[X]\). Supposons un instant que \(f\) ne soit pas identiquement nulle sur \([0,1]\) : par continuité, il existe \(0<c<1\) tel que \(f(c)\neq 0\) (et même \(f(c)>0\) quitte à remplacer \(f\) par \(-f\)) ; il existe aussi \(0<a<c<b<1\) tels \(f(x)\geq \frac{f(c)}{2},\ \forall\,x\in[a,b]\). Alors, le polynôme \(P(t)=1+(X-a)(b-X)\) vérifie \(P(t)\geq 1\) sur \([a,b]\) et \(0\leq P(t)\leq 1\) sur \(\mathbb [0,1]\setminus[a,b]\). On a aussi \(P'>0\) sur \([0,a]\) par compacité de ce dernier intervalle, il existe \(\lambda>0\) tel que \(P'(t)\geq \lambda>0\). Ainsi, en posant \(M=\sup_{[0,1]}\vert f(t)\vert\), on a pour tout \(k\in\mathbb N\) : \[\begin{aligned}\left\vert\int_0^a P^n(t)f(t)dt\right\vert&\leq M\int_0^a P^n(t)dt \leq \dfrac{1}{\lambda}\int_0^a P'(t)P^n(t)dt\\ &=\dfrac{1}{\lambda}\left[ \dfrac{P^{n+1}(t)}{n+1}\right]_0^a =\dfrac{1-(1-ab)^{n+1}}{\lambda (n+1)}\leq \dfrac{1}{\lambda(n+1)} \end{aligned}\] et en procédant de même sur \([b,1]\) on peut donc affirmer que \[\lim_{n\to\infty}\int_0^a P^n(t)f(t)dt=0,\quad\&\quad\lim_{n\to\infty}\int_b^1 P^n(t)f(t)dt=0.{\text{(\ding{56})}}\] Par contre, sur \([a,b]\), et avec ce choix de \(P\) nous avons \[\int_a^b P^n(t)f(t)dt\geq \int_a^b f(t)dt\geq \dfrac{b-a}{2}f(c)>0.{\text{(\ding{52})}}\] Les formules () et () impliquent que \(\int_0^1 P^n(t)f(t)dt>0\) pour tout entier \(n\) suffisament grand, fait parfaitement absurde puisque \(\int_0^1 Q(t)f(t)dt=0,\quad\forall\,Q\in\mathbb R[X]\).

Utilisez correctement le théorème de Weierstrass, en particulier sur \(\mathbb R^2\) ou \(\mathbb C\) toute fonction continue sur un compact \(K\) y est limite uniforme de polynômes en \(x\) et \(y\) (i.e. \(\in\mathbb C[x,y]\)) et non en \(z=x+iy\) (i.e. \(\in\mathbb C[z]\)) . Le contre exemple canonique (bien connu des amateurs de fonctions holomorphes) étant la fonction continue sur \(\mathbb C^\star\) : \(f(z)= z^{-1}\), pour laquelle il n’existe pas de suite de polynômes \((P_n)_n\subset\mathbb C[z]\) convergeant uniformément vers \(f\) sur le cercle unité \(S^1\subset\mathbb C^\star\) ; en effet en passant par exemple en coordonnées polaires on vérifie facilement que

\[\int_{S^1}f(z)dz=2i\pi\quad\text{ alors que }\quad\int_{S^1}P_n(z)dz=0,\quad\forall\,n\in\mathbb N.\]


Accordéon
Titre
Solution
Texte

Étude de \(I_\alpha = \int_0^{+\infty}{{\sin(t)}\over t^\alpha}dt, \quad J_\alpha = \int_2^{+\infty}{{\sin(t)}\over {t^\alpha+\sin(t)}} dt \quad \& \quad S_\alpha = \sum_{n\geq 1}{{(-1)^n}\over{n^\alpha + (-1)^n}},\ \alpha\in\mathbb R\)
Par Patrice Lassère le 9 novembre 2022 16:17

Notons \(f(t)=\sin(t) t^{-\alpha}\). \(I_\alpha\) est une intégrale impropre à l’origine et à l’infini. À l’origine, \(f\) est équivalente à \(t^{-\alpha+1}\) fonction positive, de type Riemann, intégrable en \(t=0\) puisque \(-\alpha+1<-1\). En outre \(\vert f(t)\vert\leq t^{-\alpha}\) fonction intégrable à l’infini pour \(\alpha>1\) : la convergence absolue de \(\mathbf{I_\alpha}\) est donc établie pour \(2>\alpha>1\).

Reste donc le cas  \(0< \alpha\leq 1\) ; vu ce qui précède (tout se passant bien à l’origine) il sera suffisant d’étudier la convergence sur \([1,+\infty[\). Pour cela on fait une intégration par parties (avec la convention habituelle qu’elle sera réellement correcte si deux termes parmi les trois existent, sinon, travailler sur \([1,A]\) puis faire tendre \(A\) vers \(+\infty\)...)

\[\int_1^{+\infty}{{\sin(t)}\over t^\alpha}dt= \left[ {{\cos(t)}\over{t^{\alpha}}} \right] _1^{+\infty}+\int_1^{+\infty} {{\alpha\cos(t)}\over{t^{\alpha+1}}}dt\]

et \[\left[ {{\cos(t)}\over{t^{\alpha}}} \right ]_1^{+\infty}=-\cos(1),\quad \left\vert {{\alpha\cos(t)}\over{t^{\alpha+1}}} \right\vert \leq {\alpha\over t^{\alpha+1}}\in L^1([1,+\infty[)\] assurent alors la convergence de \(\mathbf {I_{\alpha}}\) pour \(\mathbf{0< \alpha \leq 1}\).

Pour la convergence absolue en \(+\infty\) si \(0<\alpha\leq 1\), le plus simple est de remarquer que

\[\left\vert {\sin(t)\over t^{\alpha}} \right\vert \geq \left\vert {\sin^2(t)\over t^{\alpha}} \right\vert \geq {1\over t^\alpha}-{{\cos(2t)}\over t^\alpha} =g(t)-h(t)\]

en raisonnant comme au dessus \(h\) est intégrable en \(+\infty\) mais bien sûr \(g\) ne l’est pas, et l’intégrale \(\displaystyle\int_{\mathbb R_+} \left\vert {\sin(t)\over t^{\alpha}} \right\vert dt\) diverge pour tout \(\mathbf {0<\alpha\leq 1}\).

Il est aussi classique de conclure par la minoration

\[\int_0^{{(2n+1)\pi}} \left\vert {\sin(t)\over t^{\alpha}} \right\vert dt \geq \sum_{k=0}^n\int_{2k\pi+{\pi\over 4}}^{2k\pi+{3\pi\over 4}} \left\vert {\sin(t)\over t^{\alpha}} \right\vert dt \geq \sum_{k=0}^n\int_{2k\pi+{\pi\over 4}}^{2k\pi+{3\pi\over 4}}{{\sqrt{2}}\over{2\left((2k+1)\pi\right)^\alpha}}dt =\sum_{k=0}^n{{\pi\sqrt{2}}\over {4\left((2k+1)\pi\right)^\alpha}}\]

cette dernière quantité tendant vers l’infini avec \(n\) comme somme partielle d’une série de terme général équivalent à \(Ck^{-\alpha}\) donc divergente...

Le problème est toujours à l’infini où notre fonction n’est pas de signe constant, donc pas question d’utiliser les équivalents, le dernier recours (le développement asymptotique) montre ici toute sa puissance

\[\begin{aligned} {{\sin(t)}\over {t^\alpha+\sin(t)}} &={{\sin(t)}\over t^\alpha}\left({1\over{1+t^ {-\alpha}\sin(t)}}\right)={{\sin(t)}\over t^\alpha}\left( 1-{\sin(t)\over t^{\alpha}}+o\left({1\over t^{\alpha}}\right)\right) ={\sin(t)\over t^{\alpha}}-{\sin^2(t)\over t^{2\alpha}}+ o\left({1\over t^{\alpha}}\right)\\ &= g(t) +h(t)\qquad\text{où }\quad g(t)={\sin(t)\over t^{\alpha}}\quad\text{ et }\quad h(t)= -{\sin^2(t)\over t^{2\alpha}}+ o\left({1\over t^{\alpha}}\right)\end{aligned}\]

vu 2) la fonction \(g\) est intégrable à l’infini pour tout \(\alpha>0\), par contre \[h(t)\sim {-\sin^2(t)\over t^{2\alpha}}={1\over 2t^{2\alpha}}-{{\cos(2t)}\over 2t^{2\alpha}}\] le sera si et seulement si \(\alpha> 1/2\). La seconde intégrale est donc convergente si et seulement si \(\alpha>1/2\). Il est bien de remarquer que pour tout \(\alpha>0\) : \[{{\sin(t)}\over {t^\alpha+\sin(t)}}\sim {{\sin(t)}\over {t^\alpha}}\] la première étant intégrable si et seulement si \(\alpha>1/2\), et la seconde, si et seulement si \(\alpha>0\) :

L’intégrabilité ne passe donc pas à l’équivalent lorsque les fonctions ne sont pas de signe constant.

C’est la version série du second exemple et se traite de la même manière.

Remarques : L’étude de la série donne \[u_n={{(-1)^n}\over{n^\alpha +(-1)^n}}\sim {{(-1)^n \over {n^{\alpha}}}}\] pour tout \(\alpha>0\) et comme plus haut, pour \(0<\alpha\leq 1/2,\ \sum_n u_n\) diverge, ce qui peut donner lieu à deux remarques :

La première est un contre-exemple pour le théorème sur les équivalents lorsque les séries ne sont pas de signe constant (puisque la série de terme général \((-1)^n n^{-\alpha}\) est convergente pour tout \(\alpha>0\)).

La seconde, est que \(\sum_n u_n\) étant alternée, divergente et son terme général tendant vers zéro : \(\displaystyle v_n= \left\vert {{(-1)^n}\over{n^\alpha +(-1)^n}} \right\vert\) ne décroit pas vers zéro (vu le théorème des les séries alternées). Toutefois \(v_n\sim n^{-\alpha}\) i.e. la monotonie non plus ne passe pas à l’équivalent.


Accordéon
Titre
Solution
Texte

Une caractérisation de la fonction Gamma : le théorème de Bohr-Mollerup
Par Patrice Lassère le 9 novembre 2022 16:17

On suppose ici acquis (classique mais excellent exercice sur les intégrales à paramètres) le fait que \(\Gamma\in\mathscr C^\infty(\mathbb R_+^\star)\) et que l’on peut dériver sous l’intégrale.

Il suffit de faire \(n\) intégrations par parties en faisant tomber le degré de \((1-{t\over n})^n\).

Soit \(f_n(t)=(1-{t\over n})^nt^{x-1}\mathbb I_{[0,n]}(t)\). La suite \((f_n)_n\) est simplement convergente sur \(\mathbb R_+^\star\) vers \(g(t)=t^{x-1}e^{-t}\) et pour \(t>0,\,n\geq 1\ :\ 0\leq f_n(t)\leq g(t)\in L^1(\mathbb R_+)\). Par convergence dominée le résultat suit.

Il suffit de combiner les deux premières questions.

\(\text{log}(\Gamma)''=(\Gamma''\Gamma-{\Gamma'}^2)\Gamma^{-2}\) avec \(\Gamma^{(k)}(x)=\int_0^{+\infty}f_k(x,t)dt\)\(f_k(t)=\log^k(t)t^{x-1}e^{-t},\ k=0,1,2\). En écrivant \(f_1(x,t)= g(t)h(t)\) avec \(g(t)=\log(t)t^{(x-1)/2}e^{-t/2}\) et \(h(t)=t^{(x-1)/2}e^{-t/2}\) si on remarque que \(g\) et \(h\in L^2(\mathbb R_+)\) espace de Hilbert muni du produit scalaire \(\langle f,g\rangle =\int_{\mathbb R_+}f(t)g(t)dt\), on a par Cauchy-Schwarz :

\[(\Gamma''\Gamma-{\Gamma'}^2)(x)=\Vert g\Vert^2 \Vert h\Vert^2-\langle g,h\rangle\geq 0.\]

\(\log(\Gamma)\) est donc convexe puisque de dérivée seconde positive.

Soient \(0<x<y\) et \(0<t<1\). On a \(f(tx+(1-t)y)\leq f^t(x)f^{1-t}(y)\) et en particulier, si \(u=tx+(1-t)y\) et \(n\in\mathbb N^\star\) la formule \(f(x+1)=xf(x)\) donne \[\begin{aligned} f(u+n+1)= u(u+1) \dots (u+n)f(u)&\leq f(x+n+1)^tf(y+n+1)^{1-t}\\ &\leq \left(x(x+1)\dots (x+n)f(x)\right)^t\left(y(y+1)\dots (y+n)f(y)\right)^{1-t}\end{aligned}\] soit \[{{u(u+1) \dots (u+n)f(u)}\over{n^un!}}\leq \left({{x(x+1)\dots (x+n)f(x)}\over{n^xn!}}\right)^t\left({{y(y+1)\dots (y+n)f(y)}\over{n^yn!}}\right)^{1-t}\]

et, vu

, si \(n\) tend vers l’infini : \[{{f(tx+(1-t)y)}\over{\Gamma(tx+(1-t)y)}}\leq \left({{f(x)}\over{\Gamma(x)}}\right)^t\left({{f(y)}\over{\Gamma(y)}}\right)^ {1-t}\]

La fonction \(f/\Gamma\) est donc log-convexe continue sur \(\mathbb R_+^\star\), mais aussi \(1\)-périodique puique \(f\) est \(\Gamma\) satisfont à la même équation fonctionelle \(g(x+1)=xg(x)\) : la fonction \(\log\left(f/\Gamma\right)\) est donc à la fois convexe et bornée : elle ne peut être que constante (voir l’exercice \(3\)) et le tour est joué vu que \(f(1)=\Gamma(1)=1\).


Accordéon
Titre
Solution
Texte

Sommes de Riemann et formule de Taylor
Par Patrice Lassère le 9 novembre 2022 16:17

Commencons par observer que \[\begin{aligned} n\left(\dfrac{1}{n}\sum_{k=1}^nf\left(\dfrac{k}{n} \right)-\int_0^1f(t)dt\right)&= n\left(\dfrac{1}{n}\sum_{k=1}^nf\left(\dfrac{k}{n} \right)-\sum_{k=1}^n\int_{\frac{k-1}{n}}^{\frac{k}{n}}f(t)dt\right)\\ &= n\sum_{k=1}^n \int_{\frac{k-1}{n}}^{\frac{k}{n}}\left( f\left( \dfrac{k}{n}\right) -f(t)\right)dt \\ &=n\sum_{k=1}^n \int_{\frac{k-1}{n}}^{\frac{k}{n}}f'(\zeta_k(t))\left( \dfrac{k}{n}-t\right) dt,\\ \end{aligned}\] Et si on pose pour \(1\leq k\leq n\) \[m_k:=\inf\left\lbrace f'(x)\ :\ x\in\left[ \dfrac{k-1}{n},\dfrac{k}{n}\right] \right\rbrace\] \[M_k:=\sup\left\lbrace f'(x)\ :\ x\in\left[ \dfrac{k-1}{n},\dfrac{k}{n}\right] \right\rbrace\] on obtient \[m_k\int_{\frac{k-1}{n}}^{\frac{k}{n}}\left( \dfrac{k}{n}-t\right) dt\leq \int_{\frac{k-1}{n}}^{\frac{k}{n}}f'(\zeta_k(t))\left( \dfrac{k}{n}-t\right) dt \leq M_k\int_{\frac{k-1}{n}}^{\frac{k}{n}}\left( \dfrac{k}{n}-t\right) dt\] soit finalement \[\dfrac{1}{2n}\sum_{k=1}^n m_k\leq n\sum_{k=1}^n \int_{\frac{k-1}{n}}^{\frac{k}{n}}f'(\zeta_k(t))\left( \dfrac{k}{n}-t\right) dt \leq\dfrac{1}{2n} \sum_{k=1}^n M_k\] mais \[\dfrac{1}{2}\int_0^1f'(t)dt=\lim_{n\to\infty}\dfrac{1}{2n} \sum_{k=1}^n M_k=\lim_{n\to\infty}\dfrac{1}{2n} \sum_{k=1}^n m_k\] car on y reconnait dans les deux termes de gauche deux sommes de Riemann associées à la fonction \(f'\) sur \([0,1]\). C.Q.F.D.

Pour la seconde limite, la technique est la même mais il faut bien entendu pousser plus loin le développement : avec Taylor-Lagrange, nous avons \[f(x)-f\left( \dfrac{2i-1}{2n}\right) =f'\left( \dfrac{2i-1}{2n}\right) \left( x-\dfrac{2i-1}{2n}\right) +\dfrac{1}{2}f''\left( \zeta_i(x)\right) \left( x-\dfrac{2i-1}{2n}\right) ^2.\] Et par conséquent \[\begin{aligned} n^2\left(\int_0^1f(t)dt- \dfrac{1}{n}\sum_{k=1}^nf\left(\dfrac{2k-1}{n} \right) \right)&= n^2\sum_{i=1}^n \int_{\frac{i-1}{n}}^{\frac{i}{n}}\left( f(t)-f\left( \dfrac{2i-1}{2n}\right) \right) dt\\ &=n^2\sum_{i=1}^n \int_{\frac{i-1}{n}}^{\frac{i}{n}}f'\left( \dfrac{2i-1}{2n}\right) \left( x-\dfrac{2i-1}{2n}\right) dx\\ &\quad +\dfrac{n^2}{2}\sum_{i=1}^n \int_{\frac{i-1}{n}}^{\frac{i}{n}}f''\left( \zeta_i(x)\right) \left( x-\dfrac{2i-1}{2n}\right)^2dx. \end{aligned}\] Il suffit maintenant de remarquer que chacunes des intégrales \[\int_{\frac{i-1}{n}}^{\frac{i}{n}}f'\left( \dfrac{2i-1}{2n}\right) \left( x-\dfrac{2i-1}{2n}\right) dx\] s’annulent. Quant aux suivantes, en raisonnant comme dans la première partie \[\dfrac{1}{24n}\sum_{i=1}^n m_i\leq \dfrac{n^2}{2}\sum_{i=1}^n \int_{\frac{i-1}{n}}^{\frac{i}{n}}f''\left( \zeta_i(x)\right) \left( x-\dfrac{2i-1}{2n}\right)^2dx \leq \dfrac{1}{24n}\sum_{i=1}^n M_i\]\(M_i\) (resp. \(m_i\)) désigne la borne supérieure (resp. inférieure) de \(f''\) sur l’intervalle \(\left[ \frac{i-1}{n},\frac{i}{n}\right]\). \(f''\) étant continue on conclut rapidement comme plus haut.


Accordéon
Titre
Solution
Texte

Autour de l’inégalité de Jensen
Par Patrice Lassère le 9 novembre 2022 16:17

On procède par récurrence sur \(n\geq 2\), la propriété étant vérifiée pour \(n=2\). Soit \(n\geq 3\), et supposons la propriété vraie jusqu’au rang \(n-1\), sans perdre de généralité supposons aussi \(0<\lambda_n<1\) ; on peut alors écrire \[\sum_{j=1}^n\lambda_j x_j=\lambda_nx_n+(1-\lambda_n)\sum_{j=1}^{n-1}\dfrac{\lambda_j}{1-\lambda_n}x_j.\] Avec cette représentation, la convexité de \(f\) et l’hypothèse de récurrence assurent que \[\begin{aligned} f\left( \sum_{j=1}^n \lambda_j x_j\right)&\leq \lambda_n x_n+(1-\lambda_n)f\left( \sum_{j=1}^{n-1}\dfrac{\lambda_j}{1-\lambda_n}x_n\right) \\ &\leq \lambda_n x_n+(1-\lambda_n)\sum_{j=1}^{n-1}\dfrac{\lambda_j}{1-\lambda_n}f\left( x_n\right) \\ &=\sum_{j=1}^n \lambda_j x_j, \end{aligned}\] soit la propriété au rang \(n\). L’inégalité de Jensen est démontrée.

Supposons par l’absurde qu’il existe \(n\geq 3\), \(x_1,\dots,x_n\in I\) non tous égaux, \(\lambda_1,\dots,\lambda_n\in \mathbb R_+^\star\) de somme \(1\) tels que \[f(\lambda_1x_1+\dots+\lambda_nx_n)= \lambda_1f(x_1)+\dots+\lambda_n f(x_n).\] L’hypothèse sur les \(x_j\) assure que \[\emptyset\neq S:=\{j\in\{1,2,\dots,n\}\ :\ x_j\neq \max_{1\leq i\leq n}x_i\}\underset{\neq}{\subset}\{1,2,\dots,n\}.\] Posons \[\lambda=\sum_{j\in S}\lambda_j,\quad x=\sum_{j\in S}\dfrac{\lambda_j}{\lambda}x_j,\quad y=\sum_{j\not\in S}\dfrac{\lambda_j}{1-\lambda}x_j\] si \(x_{n_0}:=\max_{1\leq i\leq n}x_i\}\), alors \[y=\sum_{j\not\in S}\dfrac{\lambda_j}{1-\lambda}x_j=x_{n_0}\sum_{j\not\in S}\dfrac{\lambda_j}{1-\lambda}=\dfrac{x_{n_0}}{1-\lambda}\sum_{j\not\in S}{\lambda_j}=x_{n_0}\] et \[x= \sum_{j\in S}\dfrac{\lambda_j}{\lambda}x_j< x_{n_0}\sum_{j\in S}\dfrac{\lambda_j}{\lambda}=x_{n_0}=y\] \(x\) est donc different de \(y\) : par stricte convexité de \(f\) nous avons \[f(\lambda_1x_1+\dots+\lambda_nx_n)=f(\lambda x+(1-\lambda)y)<\lambda f(x)+(1-\lambda)f(y).\] Maintenant, avec l’inégalité de Jensen \[\begin{aligned} \lambda f(x)+(1-\lambda)f(y)&=\lambda f\left( \sum_{j\in S}\dfrac{\lambda_j}{\lambda}x_j\right) +(1-\lambda)f\left( \sum_{j\not\in S}\dfrac{\lambda_j}{1-\lambda}x_j\right)\\ & \leq p \sum_{j\in S}\dfrac{\lambda_j}{\lambda}f(x_j)+(1-\lambda) \sum_{j\not\in S}\dfrac{\lambda_j}{1-\lambda}f(x_j)\\ &=\sum_{j=1}^n\lambda_j f(x_j) \end{aligned}\] les deux extrémités de ces inégalités nous donnent \[f(\lambda_1x_1+\dots+\lambda_nx_n)<\sum_{j=1}^n\lambda_j f(x_j)\] d’où la contradiction désirée.

\(\varphi\) étant convexe sur \([m,M]\), nous avons \[\varphi\left( \sum_{i=1}^n\dfrac{1}{n}f\left( a+k\dfrac{b-a}{n}\right) \right) \leq \sum_{i=1}^n\dfrac{1}{n}\varphi\left( f\left( a+k\dfrac{b-a}{n}\right) \right).\] Il ne reste plus qu’à faire tendre \(n\) vers \(+\infty\) en invoquant la continuité de \(\varphi\) et les sommes de Riemann.


Limite en \(0\), \(\pm\infty\) de \(\left(\int_a^b\vert f(t)\vert^p dt\right)^{1/p}\) *

9 novembre 2022 16:17 — Par Patrice Lassère

Montrer que si \(f\) est continue sur \([a,b]\), alors

\[\lim_{p\to+\infty}\left(\int_a^b\vert f(t)\vert^p dt\right)^{1/p} =\max_{x\in[a,b]}\vert f(x)\vert.\]

si de plus \(f\) est sans zéros dans \([a,b]\) déterminer les limites suivantes

\[\lim_{p\to 0}\left(\int_a^b\vert f(t)\vert^p dt\right)^{1/p} \quad\&\quad \lim_{p\to-\infty}\left(\int_a^b\vert f(t)\vert^p dt\right)^{1/p}.\]



[ID: 2805] [Date de publication: 9 novembre 2022 16:17] [Catégorie(s): Intégration ] [ Nombre commentaires: 0] [nombre d'éditeurs: 1 ] [Editeur(s): Emmanuel Vieillard-Baron ] [nombre d'auteurs: 1 ] [Auteur(s): Patrice Lassère ]
Accordéon
Titre
Solution
Texte

Limite en \(0\), \(\pm\infty\) de \(\left(\int_a^b\vert f(t)\vert^p dt\right)^{1/p}\)
Par Patrice Lassère le 9 novembre 2022 16:17

Il existe \(c\in[a,b]\) tel que \(M:=\vert f(c)\vert=\max_{x\in[a,b]}\vert f(x)\vert\), on a immédiatement

\[\left(\int_a^b\vert f(t)\vert^p dt\right)^{1/p}\leq \left(\int_a^bM^p dt\right)^{1/p} = M(b-a)^{1/p},{(\bigstar)}\]

De l’autre côté, par continuité de \(f\) et étant donné \(\varepsilon>0\), il existe \([\alpha,\beta]\subset[a,b]\) tel que \(\vert f(x)\vert\geq M-{\varepsilon\over 2}\) sur \([\alpha,\beta]\). Alors

\[\left(\int_a^b\vert f(t)\vert^p dt\right)^{1/p} \geq \left(\int_\alpha^\beta\vert f(t)\vert^p dt\right)^{1/p}\geq (M-{\varepsilon\over 2})(\beta-\alpha)^{1/p}\]

Ainsi, vu \((\bigstar)\), on a pour tout \(\varepsilon>0\) :

\[(M-{\varepsilon\over 2})(\beta-\alpha)^{1/p} \leq \left(\int_a^b\vert f(t)\vert^p dt\right)^{1/p} \leq M(b-a)^{1/p}\]

d’où le résultat en faisant tendre \(p\) vers \(+\infty\) (\(\varepsilon>0\) étant arbitraire).

De la première limite on déduit aussitôt (si \(f\) est sans zéros) que

\[\lim_{p\to-\infty}\left(\int_a^b\vert f(t)\vert^p dt\right)^{1/p} =\min_{x\in[a,b]}\vert f(x)\vert.\]

Reste la limite en zéro. On peu écrire

\[\log\left( {\left(\int_a^b\vert f(t)\vert^p dt\right)^{1/p}\over b-a}\right) ={1\over p}\log\left({\int_a^b\vert f(t)dt\vert^p\over b-a} \right)={1\over p}\log\left({\int_a^b\,g(t,p)dt\over b-a}\right):={h(p)-h(0)\over p}\]

vu les théorèmes classiques sur la continuité et la dérivabilité des intégrales à paramètres on vérifie sans peine que \(h\) est dérivable à l’origine, on peut dériver sous l’intégrale, si bien que

\[\lim_{p \to 0}\log\left( {\left(\int_a^b\vert f(t)\vert^p dt\right)^{1/p}dt\over b-a}\right) = h'(0) = \int_a^b\,\log(f(t)) dt.\]


Étude de la suite \((\int_0^\infty n\log\left(1+n^{-\alpha}f^\alpha (t)\right)dt)_n\) *

9 novembre 2022 16:17 — Par Patrice Lassère

(
[rms], 1999/00).

Pour \(f\in L^1(\mathbb R_+),\ \alpha\geq 1,\ n\geq 1\) on pose

\[I_n:=\int_0^\infty n\log\left(1+n^{-\alpha}f^\alpha (t)\right)dt.\]

Après avoir justifié l’existence de \(I_n\), étudier la convergence de la suite \((I_n)_n\).



[ID: 2807] [Date de publication: 9 novembre 2022 16:17] [Catégorie(s): Intégration ] [ Nombre commentaires: 0] [nombre d'éditeurs: 1 ] [Editeur(s): Emmanuel Vieillard-Baron ] [nombre d'auteurs: 1 ] [Auteur(s): Patrice Lassère ]
Accordéon
Titre
Solution
Texte

Étude de la suite \((\int_0^\infty n\log\left(1+n^{-\alpha}f^\alpha (t)\right)dt)_n\)
Par Patrice Lassère le 9 novembre 2022 16:17

Notons \(\varphi_n(t)= n\log\left(1+n^{-\alpha}f^\alpha (t)\right)\), remarquons que pour tout \(\alpha\geq 1\) et \(t\geq 0\) :

\[\ 1+t^\alpha\leq (1+t)^\alpha,\]

ainsi

\[0\leq \varphi_n(t)\leq n\alpha\log\left(1+n^{-1}f(t)\right) \leq \alpha f(t)\in L^1(\mathbb R_+){(\text{\ding{56}})}\]

la dernière inégalité résultant de l’archi-classique \(\log(1+u)\leq u\) sur \(\mathbb R_+\) i.e. pour tout \(n\in\mathbb N^\star, \varphi_n\in L^1(\mathbb R_+)\) et notre suite est bien définie.

Soit \(x\in\mathbb R_+\) fixé, si \(f(x)\ne 0\) alors \(\varphi_n(x)\sim n^{1-\alpha}f^\alpha(t)\rightarrow 0\) pour \(n\to\infty\) si \(\alpha>1\). Ainsi pour \(\alpha>1\) la suite \((\varphi_n)_n\) converge simplement sur \(\mathbb R_+\) vers la fonction nulle. Pour \(\alpha=1\) elle est simplement convergente sur \(\mathbb R_+\) vers \(f\). Ceci et \((\text{\ding{56}})\) nous permet dans les deux cas d’appliquer le théorème de la convergence dominée, soit finallement

\[\lim_{n\to\infty}I_n= \begin{cases} 0&\text{ si }\alpha>1 \\ \int_0^\infty f(t)dt &\text{ si }\alpha = 1.\end{cases}\]


Accordéon
Titre
Solution
Texte

Le lemme de Cantor
Par Patrice Lassère le 9 novembre 2022 16:17

Avec \(x=0\) on a déja \(\lim_n a_n=0\). Vu l’hypothèse, on a alors \(\lim_n b_n\sin(nx)=0,\ \forall\,x\in\mathbb R\) ; supposons que \((b_n)_n\) ne converge pas vers zéro : il existe alors \(\varepsilon>0\), une suite strictement croissante d’entiers \((n_k)_k\) vérifiant pour tout \(k\) : \(\vert b_{n_k}\vert\geq \varepsilon\). Mais alors, puisque \(\left(b_{n_k}\sin(n_k x)\right)_k\) converge simplement vers zéro sur \(\mathbb R\) la suite \(\left(\sin(n_k x)\right)_k\) est nécessairement simplement convergente vers zéro sur \(\mathbb R\), en outre \(0\leq \sin^2(n_k x)\leq 1\in L^1([0,2\pi])\). On peut donc appliquer le théorème de la convergence dominée :

\[\lim_k\int_0^{2\pi}\sin^2 (n_k x)dx = 0,\]

cependant, d’un autre côté on a aussi

\[\int_0^{2\pi}\sin^2 (n_k x)dx=\int_0^{2\pi}\left(1-\cos(2n_k x)\right){dx\over 2}=\pi\]

d’où la contradiction et \(\lim_n b_n=0.\)

Cas général : Supposons maintenant la convergence simple seulement sur un intervalle \([a,b]\). Si par exemple la suite \((a_k)_k\) ne converge pas vers \(0\), on considère comme dans la première partie \(\varepsilon>0\) et une sous-suite \((a_{n_k})_k\) tels que \(\vert a_{n_k}\vert\geq \varepsilon,\ \forall\,k\in\mathbb N\). Alors pour tout \(x\in [a,b]\) : \[\vert f_{n_k}(x)\vert:=\left\vert\dfrac{(a_{n_k}\cos(n_k x)+b_{n_k}\sin(n_k x))^2}{a_{n_k}^2+b_{n_k}^2}\right\vert\leq \left\vert\dfrac{(a_{n_k}\cos(n_k x)+b_{n_k}\sin(n_k x))^2}{\varepsilon^2}\right\vert.\] et la suite \((f_{n_k})_k\) est simplement convergente sur \([a,b]\). En outre, vu l’inégalité (Cauchy-Schwarz par exemple) \(ax+by\leq \sqrt{a^2+b^2}\sqrt{x^2+y^2}\) nous avons aussi la domination : \[\vert f_{n_k}(x)\vert\leq \dfrac{a_{n_k}^2+b_{n_k}^2}{a_{n_k}^2+b_{n_k}^2}=1,\quad\forall\,x\in[a,b],\ k\in\mathbb N.\] Donc, par convergence dominée \[\lim_{k\to\infty}\int_a^b f_{n_k}(x)dx=0{(\bigstar)}.\] Mais par un calcul direct \[\begin{aligned} \int_a^b f_{n_k}(x)dx&=\dfrac{1}{2(a_{n_k}^2+b_{n_k}^2)}\int_a^b \left( a_{n_k}^2(1+\cos(2n_kx))+b_{n_k}^2(1-\cos(2n_k x))+2a_{n_k}b_{n_k}\sin(2n_kx)\right) dx\\ &=\dfrac{b-a}{2}+ \dfrac{a_{n_k}^2-b_{n_k}^2}{2(a_{n_k}^2+b_{n_k}^2)}\int_a^b\cos(2n_kx)dx+ \dfrac{a_{n_k}b_{n_k}}{a_{n_k}^2+b_{n_k}^2}\int_a^b\sin(2n_kx)dx\\ &:=\dfrac{b-a}{2}+ \dfrac{a_{n_k}^2-b_{n_k}^2}{2(a_{n_k}^2+b_{n_k}^2)}I_k+ \dfrac{a_{n_k}b_{n_k}}{a_{n_k}^2+b_{n_k}^2}J_k \end{aligned}\] mais bien entendu \[\lim_{k\to\infty}I_k =\lim_{k\to\infty}J_k=0\] et \[\left\vert\dfrac{a_{n_k}^2-b_{n_k}^2}{2(a_{n_k}^2+b_{n_k}^2)}\right\vert\leq \dfrac{1}{2}\quad\text{et}\quad \left\vert\dfrac{a_{n_k}b_{n_k}}{a_{n_k}^2+b_{n_k}^2}\right\vert\leq\dfrac{1}{2}\] donc \[\lim_{k\to\infty}\left\vert\dfrac{a_{n_k}^2-b_{n_k}^2}{2(a_{n_k}^2+b_{n_k}^2)}I_k\right\vert+ \left\vert\dfrac{a_{n_k}b_{n_k}}{a_{n_k}^2+b_{n_k}^2}J_k\right\vert \leq \dfrac{1}{2}\left( \vert I_k\vert+\vert J_k\vert\right) =0\] soit \[\lim_{k\to\infty}\int_a^b f_{n_k}(x)dx=\dfrac{b-a}{2}>0\] contredisant \((\bigstar)\) : l’affaire est donc entendue (on procède de même si c’est la suite \((b_n)_n\) ne converge pas vers zéro).


Étude de \(x\mapsto\int_x^{x^2}\frac{dt}{\log(t)}\) *

9 novembre 2022 16:17 — Par Patrice Lassère

On définit \(F\ :\ ]1,+\infty[\to\mathbb R\) par \[F(x)=\int_x^{x^2}\frac{dt}{\log(t)}.\] Montrer que \(F\) est injective et préciser son image \(F(]1,+\infty[)\).



[ID: 2811] [Date de publication: 9 novembre 2022 16:17] [Catégorie(s): Intégration ] [ Nombre commentaires: 0] [nombre d'éditeurs: 1 ] [Editeur(s): Emmanuel Vieillard-Baron ] [nombre d'auteurs: 1 ] [Auteur(s): Patrice Lassère ]
Accordéon
Titre
Solution
Texte

Étude de \(x\mapsto\int_x^{x^2}\frac{dt}{\log(t)}\)
Par Patrice Lassère le 9 novembre 2022 16:17

\(F\) est clairement dérivable sur \(]1,+\infty[\) avec \[F'(x)=\dfrac{x-1}{\log(x)},\quad\forall\,x\in ]1,+\infty[.\] soit \(F'(x)>0\) sur \(]1,+\infty[\) : \(F\) est strictement croissante sur \(]1,+\infty[\) et en particulier injective. Comme \[F(x)\geq (x^2-x)\rm{min}\left\lbrace \,\dfrac{1}{\log(t)}\ :\ x\leq t\leq x^2\right\rbrace =\dfrac{x^2-x}{\log(x^2)}\underset{x\to+\infty}{\longrightarrow}+\infty,\] nous avons \(F(]1,+\infty[)=]F(1_-),+\infty[\). Pour déterminer \(F(1_-)\), le changement \(t=e^u\) donne \[F(x)=\int_{\log(x)}^{2\log(x)}\dfrac{e^u}{u}du\] soit \[x\log(2)=e^{\log(x)}\int_{\log(x)}^{2\log(x)}\dfrac{du}{u}< F(x)<e^{2\log(x)}\int_{\log(x)}^{2\log(x)}\dfrac{du}{u}=x^2\log(2)\] qui assure \(F(1_-)=\log(2)\) : ainsi \(F(]1,+\infty[)=]\log(2),+\infty[\).


Optimisation et convexité *

9 novembre 2022 16:18 — Par Patrice Lassère

Soit \(f\in\mathscr C^2(\mathbb R,\mathbb R)\). Si \(f''>0\) sur \([a,b]\), déterminer la fonction affine \(l(x)=\alpha x+\beta\leq f(x)\) sur \([a,b]\) minimisant \(\displaystyle \int_a^b(f(t)-l(t))dt.\)



[ID: 2813] [Date de publication: 9 novembre 2022 16:18] [Catégorie(s): Intégration ] [ Nombre commentaires: 0] [nombre d'éditeurs: 1 ] [Editeur(s): Emmanuel Vieillard-Baron ] [nombre d'auteurs: 1 ] [Auteur(s): Patrice Lassère ]
Accordéon
Titre
Solution
Texte

Optimisation et convexité
Par Patrice Lassère le 9 novembre 2022 16:18

Il est clair que \(l(x)=f(x)\) pour au moins une valeur de \(x\) (sinon, ou pourrait déplacer le graphe de \(l\) verticalement ce qui réduirait la valeur de l’intégrale). Le graphe de \(l\) est donc tangent à celui de \(f\) et il existe \(c\in[a,b]\) tel que \[l(x)= f(c)+f'(c)(x-c)\] On peut maintenant aussi remarquer que minimiser \[\int_a^b(f(t)-l(t))dt=\int_a^bf(t)dt-\int_a^b l(t)dt\] équivaut à maximiser \[\begin{aligned}\int_a^b l(t)dt&=\int_a^b (f'(c)t+f(c)-cf(c))dt\qquad a\leq c\leq b\\ &=\dfrac{f'(c)}{2}(b^2-a^2)+(b-a)(f(c)-cf'(c))\\ &=(b-a)\left( \dfrac{f'(c)}{2}(a+b)+f(c)-cf'(c)\right). \end{aligned}\] Il s’agit donc de maximiser sur l’intervalle \([a,b]\) l’application \[g(c)=\dfrac{f'(c)}{2}(a+b)+f(c)-cf'(c).\] Or, \(g'(c)=\dfrac{f''(c)}{2}(a+b-2c)\), mais \(f''>0\) : \(g'\) est donc du signe de \(c\mapsto a+b-2c\). Ainsi \(g\) est croissante sur \([a,(a+b)/2]\) et décroissante sur \([(a+b)/2,b]\) son maximum est donc atteint au point médian \((a+b)/2\). L’inf de notre problème est donc réalisé par la tangente à la courbe au point \((a+b)/2\) : \[l(x)=f\left( \dfrac{a+b}{2}\right) +f'\left( \dfrac{a+b}{2}\right)\left( x-\dfrac{a+b}{2}\right).\]


L’inégalité de Hardy \(\int_0^T\left(x^{-1}\int_0^xf(u)du\right)^2dx\leq 4\int_0^Tf^2(x)dx.\) *

9 novembre 2022 16:18 — Par Patrice Lassère

[rms]-2006.

Soit \(f\in\mathscr C(\mathbb R_+,\mathbb R)\cap L^2(\mathbb R_+)\). Pour \(x>0\) on pose \[g(x)= \dfrac{1}{x}\int_0^x\,f(t)dt.\] Montrer que \(g\in L^2(\mathbb R_+)\) et trouver une constante absolue \(C>0\) telle que \[\int_0^\infty\,g^2(t)dt\leq 4\int_0^\infty\,f^2(t)dt.\]



[ID: 2815] [Date de publication: 9 novembre 2022 16:18] [Catégorie(s): Intégration ] [ Nombre commentaires: 0] [nombre d'éditeurs: 1 ] [Editeur(s): Emmanuel Vieillard-Baron ] [nombre d'auteurs: 1 ] [Auteur(s): Patrice Lassère ]
Accordéon
Titre
Solution
Texte

L’inégalité de Hardy \(\int_0^T\left(x^{-1}\int_0^xf(u)du\right)^2dx\leq 4\int_0^Tf^2(x)dx.\)
Par Patrice Lassère le 9 novembre 2022 16:18

\(\bullet\)\(g\) est continue sur \(\mathbb R_+^\star\) et pour \(x>0\) on a avec Cauchy-Schwarz \[\vert g(x)\vert=\dfrac{1}{x}\left\vert\int_0^x\,f(t)dt\right\vert \leq\dfrac{1}{x}\left(\int_0^x dt\right)^{1/2}\left(\int_0^x f^2(t)dt\right)^{1/2}\\ = \dfrac{\left(\int_0^x f^2(t)dt\right)^{1/2}}{\sqrt{x}}\] comme \(f\in L^2(\mathbb R_+\), il en résulte \[g(x)=o\left(\dfrac{1}{\sqrt{x}}\right),\quad\text{pour\ }x\to 0.{\text{(\ding{56})}}\] \(g\) est intégrable à l’origine et donc sur \([0,A]\) pour tout \(A>0\). On utilisera ultérieurement cette estimation.

\(\bullet\)Soit \(A>0\), on va commencer par montrer l’inégalité sur \([0,A]\) i.e. \(\int_0^A\,g^2(t)dt\leq 4\int_0^A\,f^2(t)dt\) ; on peut bien entendu supposer \(\int_0^A f(t)dt\neq 0\) sinon il n’y a rien à démonter. Soit \(0<x<A\) : \[\begin{aligned} \int_0^A\,\dfrac{1}{x^2}\left(\int_0^x\,f(t)dt\right)^2dx&= -\int_0^A\,\left(\dfrac{1}{x}\right)'\left(\int_0^x\,f(t)dt\right)^2dx\\ &=-\left[ \dfrac{1}{x}\left(\int_0^x\,f(t)dt\right)^2\right]_0^A +\int_0^A\dfrac{2f(x)}{x}\int_0^x\,f(t)dtdx\\ &=\int_0^A\dfrac{2f(x)}{x}\int_0^x\,f(t)dtdx-\dfrac{1}{A}\left(\int_0^A\,f(t)dt\right)^2\\ &\leq 2\int_0^A\dfrac{f(x)}{x}\int_0^x\,f(t)dtdx,\quad\text{car\ }\ \dfrac{1}{A}\left(\int_0^A\,f(t)dt\right)^2\geq 0, \end{aligned}\] où l’intégration par partie dans l’intégrale généralisée est légitime car deux termes parmi les trois existent, le terme en crochets étant nul à l’origine vu (). Maintenant de cette inégalité et toujours par Cauchy-Schwarz \[\begin{aligned} \int_0^A\,\dfrac{1}{x^2}\left(\int_0^x\,f(t)dt\right)^2dx&\leq 2\int_0^A\,f(x)\times\left(\dfrac{\int_0^x\,f(t)dt}{x}\right)dx\\ &\leq 2\left(\int_0^A f^2(t)dt\right)^{1/2}\left(\int_0^A\,\dfrac{1}{x^2}\left(\int_0^x\,f(t)dt\right)^2dx\right)^{1/2}\\ \end{aligned}\] soit puisque \(\int_0^A f(t)dt\neq 0\) \[\int_0^A\,g^2(t)dt=\int_0^A\,\dfrac{1}{x^2}\left(\int_0^x\,f(t)dt\right)^2dx\leq 4\int_0^A\,f^2(t)dt.{\text{(\ding{52})}}\]

\(\bullet\)Montrons que la constante \(4\) dans () est optimale. Pour cela, si elle est vérifiée pour un \(C<4\) on l’applique à \(f(t)=t^{-\alpha},\ -1/2<\alpha<1/2\) \[\int_0^A\dfrac{dx}{x^{2}}\left(\int_0^x\,\dfrac{dt}{t^\alpha}\right)^2dx =\dfrac{A^{1-2\alpha}}{(1-2\alpha)(1-\alpha)^2}\leq C\int_0^A\dfrac{dt}{t^\alpha}=C\cdot\dfrac{A^{1-2\alpha}}{1-2\alpha}\] qui donne \[\dfrac{1}{(1-\alpha)^2}\leq C,\] et en faisant tendre \(\alpha\) vers \(1/2\) on obtient \(C\geq 4\) : \(4\) est bien optimale dans ().

\(\bullet\)On a donc pour tout \(A>0\) \[0\leq \int_0^A\,g^2(t)dt\leq 4\int_0^A\,f^2(t)dt\leq 4\int_0^\infty\,f^2(t)dt<\infty,\] \(g^2\) est donc intégrable sur \(\mathbb R_+\) et \[\int_0^\infty\,g^2(t)dt\leq 4\int_0^\infty\,f^2(t)dt.\] La constante \(4\) est bien entendu encore optimale (reprendre le raisonnement précedent avec des fonctions continues sur \(\mathbb R_+^\star\) nulles sur \([2,+\infty[\) et égales à \(t^{-\alpha}\) sur \(]0,1]\) avec \(\vert\alpha\vert< 1/2\)).


Une formule de Ramanujan et le d.s.e. de la fonction tangente *

9 novembre 2022 16:18 — Par Patrice Lassère

Pour tout complexe \(z\in\mathbb C\) de partie réelle \(\vert\text{re}(z):=\sigma\vert<1\), on considère l’application \[f(z)=\begin{cases}\dfrac{\text{sh}(zt)}{\text{sh}(t)},\quad&\text{si}\ t>0,\\ \ \ z,\quad&\text{si}\ t=0.\end{cases}\]

Montrer que \(f\in\mathscr C^0(\mathbb R_+)\cap L^1(\mathbb R_+)\) et vérifie \[\int_0^{+\infty}\,f(t)dt=\sum_{n\geq 0}\dfrac{2z}{(2n+1)^2-z^2}.\]

Sachant que
1 \[\pi\text{cotan}(\pi z)=\dfrac{1}{z}+\sum_{n\geq 1}\dfrac{2z}{n^2-z^2},\quad\forall\,z\in\mathbb C\setminus\mathbb Z,\] montrer que \[\int_0^{+\infty}\,f(t)dt=\dfrac{\pi}{2}\text{tan}(\dfrac{\pi z}{2}),\quad \forall\,z\in\mathbb C,\ \vert \text{re}(z)\vert<1.\]

En développant \(f\) en série entière (pour la variable \(z\)) montrer que \[\dfrac{\pi}{2}\tan(\pi z/2)=\int_0^{+\infty}\,\dfrac{\text{sh}(zt)}{\text{sh}(t)}dt=2\sum_{`\geq 0}\left( 1-2^{-2n-2}\right)\zeta(2n+2)z^{2n+1},\quad\forall\,\vert z\vert<1.\]




[ID: 2817] [Date de publication: 9 novembre 2022 16:18] [Catégorie(s): Intégration ] [ Nombre commentaires: 0] [nombre d'éditeurs: 1 ] [Editeur(s): Emmanuel Vieillard-Baron ] [nombre d'auteurs: 1 ] [Auteur(s): Patrice Lassère ]
Accordéon
Titre
Solution
Texte

Une formule de Ramanujan et le d.s.e. de la fonction tangente
Par Patrice Lassère le 9 novembre 2022 16:18

Comme \(\lim_{t\to 0_+}f(t)=z=f(0)\), \(f\) est continue à l’origine et donc sur \(\mathbb R_+\). En outre \(\vert\sigma\vert<1\) implique \[\vert f(t)\vert =\left\vert\dfrac{e^{zt}-e^{-zt}}{2\text{sh}(t)}\right\vert\leq \dfrac{e^{\vert \sigma t\vert}}{\vert \text{sh}(t)\vert}\underset{+\infty}{\sim} 2e^{(\vert \sigma\vert-1)t}\in L^1(\mathbb R_+)\] ainsi \(f\in L^1(\mathbb R_+)\).

Pour \(t>0\), on peut écrire \[f(t)=\text{sh}(zt)\dfrac{2e^{-t}}{1-e^{-2t}}=2e^{-t}\text{sh}(zt)\sum_{n\geq 0}\,e^{-2nt}=\sum_{n\geq 0}\,\left( e^{(z-2n-1)t}-e^{-(z+2n+1)t}\right):=\sum_{n\geq 0}u_n(t),\] et les fonctions \(u_n\) sont intégrables sur \(\mathbb R_+\) car (toujours car \(\vert\sigma\vert<1\)) \[\vert u_n(t)\vert\leq e^{-2nt}\left\vert e^{(z-1)t}-e^{-(z+1)t}\right\vert\leq 2e^{-2nt}e^{(\vert\sigma\vert-1)t}\in L^1(\mathbb R_+).\] Un petit calcul nous donne \[\int_0^{+\infty}\,u_n(t)dt=\dfrac{2z}{(2n+1)^2-z^2}\] (bien remarquer que \(\vert\sigma\vert<1\) assure que \((2n+1)^2-z^2\neq 0,\ \forall\,n\in\mathbb N\)). Ainsi, formellement \[\int_0^{+\infty}\,f(t)dt=\int_0^{+\infty}\,\sum_{n\geq 0}u_n(t)dt=\sum_{n\geq 0}\int_0^{+\infty}\,u_n(t)dt =\sum_{n\geq 0}\dfrac{2z}{(2n+1)^2-z^2}\] il ne reste donc plus qu’à justifier l’échange \(\int\sum=\sum\int\) ci-dessus. Pour cela, il faut être délicat car la condition suffisante la série \(\sum_n\int_{\mathbb R_+}\vert u_n(t)\vert dt\) converge n’est pas ici réalisée1 et il faut se ramener au cadre des suites de fonctions en posant \[S_n(t)=\sum_{k=0}^n u_k(t),\quad n\in\mathbb N.\] La suite \((S_n)_n\) est simplement convergente sur \(\mathbb R_+^\star\) et vérifie \[\vert S_n(t)\vert\leq \vert\text{sh}(zt)\vert\sum_{k=0}^n e^{-(2k+1)t}\leq\vert\text{sh}(zt)\vert\sum_{k\geq 0} e^{-(2k+1)t}=\vert f(t)\vert\in L^1(\mathbb R_+).\] L’hypothèse de domination est donc vérifiée et l’échange \(\int\sum=\sum\int\) est justifié, CQFD.

Il suffit d’un peu de patience et d’attention. Avec la question précédente nous avons pour \(\vert\text{re}(z)\vert<1\) et \(z\neq 0\) (si \(z=0\) il n’y a rien à démontrer) \[\begin{aligned}\int_0^{+\infty}\,f(t)dt&=\sum_{n\geq 0}\dfrac{2z}{(2n+1)^2-z^2} =\sum_{n\geq 0}\dfrac{2z}{n^2-z^2}-\sum_{n\geq 0}\dfrac{2z}{(2n)^2-z^2}\\ &=-\dfrac{2}{z}+\sum_{n\geq 1}\dfrac{2z}{n^2-z^2}+\dfrac{2}{z}-\sum_{n\geq 1}\dfrac{2z}{(2n)^2-z^2}\\ &=-\dfrac{1}{z}-\sum_{n\geq 1}\dfrac{2z}{z^2-n^2}+\dfrac{1}{z}+\sum_{n\geq 1}\dfrac{z/2}{(z/2)^2-n^2}\\ &=-\pi\text{cotan}(\pi z)+\dfrac{1}{2}\left( \dfrac{2}{z}+\sum_{n\geq 1}\dfrac{z}{(z/2)^2-n^2}\right)\\ &=-\pi\text{cotan}(\pi z)+\dfrac{\pi}{2}\text{cotan}(\pi z/2)\\ &=\dfrac{\pi}{2}\cdot\dfrac{\cos^2(\pi z/2)-\cos(\pi z)}{\cos(\pi z/2)\sin(\pi z/2)}\\ &=\dfrac{\pi}{2}\cdot\dfrac{\sin^2(\pi z/2)}{\cos(\pi z/2)\sin(\pi z/2)}=\dfrac{\pi}{2}\tan(\pi z/2). \end{aligned}\]

Pour \(t\geq 0\) on a \[f(t)=\dfrac{\text{sh}(zt)}{\text{sh}(t)}=\sum_{n\geq 0}\dfrac{t^{2n+1}z^{2n+1}}{\text{sh}(t)(2n+1)!}:=\sum_{n\geq 0}h_n(t).\] Les applications \(h_n\) sont clairement dans \(L^1(\mathbb R_+)\) et nous avons \[\begin{aligned}\int_0^\infty\,h_n(t)dt&=\int_0^\infty\,\dfrac{t^{2n+1}z^{2n+1}}{\text{sh}(t)(2n+1)!}dt\\ &=\dfrac{2z^{2n+1}}{(2n+1)!}\int_0^\infty\,\sum_{k\geq 0}t^{2n+1}e^{-(2k+1)t}dt=\dfrac{2z^{2n+1}}{(2n+1)!}\int_0^\infty\,\sum_{k\geq 0}g_{n,k}(t)dt\\ &\underset{\text{\ding{56}}}{=}\dfrac{2z^{2n+1}}{(2n+1)!}\sum_{k\geq 0}\int_0^\infty\,t^{2n+1}e^{-(2k+1)t}dt \end{aligned}\] il s’agit bien entendu de justifier l’échange \(\int\sum=\sum\int\) signalé par le symbole : les fonctions \(g_{n,k}\) étant postives, il est suffisant de montrer que la série \(\sum_k\int_{\mathbb R_+}g_{n,k}(t)dt\) converge ce qui se vérifie facilement car aprés \(2n+1\) intégrations par parties nous avons \[\int_0^\infty\,g_{n,k}(t)dt=\int_0^\infty\,t^{2n+1}e^{-(2k+1)t}dt=\dfrac{(2n+1)!}{(2k+1)^{2n+2}},\] (les termes entre crochets étant nuls). Ainsi, pour tout \(n\in\mathbb N\) \[\begin{aligned}\int_0^\infty\,h_n(t)dt&=\dfrac{2z^{2n+1}}{(2n+1)!}\sum_{k\geq 0}\int_0^\infty\,t^{2n+1}e^{-(2k+1)t}dt=\dfrac{2z^{2n+1}}{(2n+1)!}\sum_{k\geq 0}\dfrac{(2n+1)!}{(2k+1)^{2n+2}}\\ &= 2z^{2n+1}\left(\sum_{k\geq 1}\dfrac{1}{k^{2n+2}}-\sum_{k\geq 1}\dfrac{1}{(2k)^{2n+2}}\right)\\ &=2z^{2n+1}\left(1-2^{-(2n+2)}\right)\zeta(2n+2) \end{aligned}\] Soit \[\int_0^\infty\,\vert h_n(t)\vert dt\leq 2\vert z\vert^{2n+1}\zeta(2)\] qui est pour \(\vert z\vert<1\) le terme général d’une série convergente, ceci justifie l’échange \(\int_{\mathbb R_+}\sum_n h_n=\sum_n\int_{\mathbb R_+} h_n\), et finalement \[\int_0^\infty\,\dfrac{\text{sh}(zt)}{\text{sh}(t)}dt=\sum_{n\geq 0}\,2\left(1-2^{-(2n+2)}\right)\zeta(2n+2)\cdot z^{2n+1},\quad\forall\,\vert z\vert<1.\] En particulier, avec la question précédente on a \[\dfrac{\pi}{2}\tan(\pi z/2)=\sum_{n\geq 0}\,2\left(1-2^{-(2n+2)}\right)\zeta(2n+2)\cdot z^{2n+1},\quad\forall\,\vert z\vert<1,\] c’est le développement en série entière de la fonction \(D(0,1)\ni z\mapsto \dfrac{\pi}{2}\tan(\pi z/2)\).

Commentaire : Il existe bien sûr des méthodes plus élémentaires pour déterminer le développement en série entière de la fonction tangente à l’origine, c’est ici juste une conséquence de la formule de Ramanujan \(\displaystyle\int_0^\infty\,\dfrac{\text{sh}(zt)}{\text{sh}(t)}dt=\dfrac{\pi}{2}\tan(\pi z/2)\)


  1. 1  un calcul élémentaire mais assez fastidieux donne \(\int_{\mathbb R_+}\vert u_n(t)=.........\)

\(\Gamma'(1)=-\gamma\) *

9 novembre 2022 16:18 — Par Patrice Lassère

Montrer que \[\Gamma'(1)=\lim_{n\to\infty}\int_0^n\,\left(1-\dfrac{t}{n}\right)^n\log(t)dt\] pour en déduire que \[\Gamma'(1)=-\gamma:=\lim_{n\to\infty}\left(1+\dfrac{1}{2}+\dots+\dfrac{1}{n}-\log(n)\right).\] (\(\gamma\) est la trés célèbre constante d’Euler) On pourra effectuer le changement de variables \(u=1-t/n\) dans l’intégrale.



[ID: 2819] [Date de publication: 9 novembre 2022 16:18] [Catégorie(s): Intégration ] [ Nombre commentaires: 0] [nombre d'éditeurs: 1 ] [Editeur(s): Emmanuel Vieillard-Baron ] [nombre d'auteurs: 1 ] [Auteur(s): Patrice Lassère ]
Accordéon
Titre
Solution
Texte

\(\Gamma'(1)=-\gamma\)
Par Patrice Lassère le 9 novembre 2022 16:18

Il est considéré comme acquis1 que la fontion \(\Gamma\) est de classe \(C^\infty\) sur \(\mathbb R_+^\star\) avec \[\Gamma^{(k)}(x)=\int_0^\infty\,\log^k(t)e^{-t}t^{x-1}dt,\quad\forall\,k\in\mathbb N.\]

Considérons pour tout \(n\in\mathbb N\) les fonctions \[f_n(x)=\log(t)\left(1-\dfrac{t}{n}\right)^n{\bf{1}}_{[0,n]}(t)=\begin{cases} \log(t)\left(1-\dfrac{t}{n}\right)^n \quad &\text{pour}\quad t\in[0,n],\\ 0\quad& \text{si}\quad x>n.\end{cases}\] La suite \((f_n)_n\) est simplement convergente sur \(\mathbb R_+\) vers \(t\mapsto \log(t)e^{-t}\) et l’inégalité classique \((1-t)^a\leq e^{-at}, a>0, 0\leq t\leq 1\), assure la domination \[\vert f_n(t)\vert\leq \vert\log(t)\vert e^{-t}\in L^1(\mathbb R_+).\] Le théorème de la convergence dominée peut donc s’appliquer : \[\lim_n\int_0^n\, \left(1-\dfrac{t}{n}\right)^n\log(t)dt=\lim_n\int_{\mathbb R_+}\,f_n(t)dt=\int_{\mathbb R_+}\,\log(t)e^{-t}dt=\Gamma'(1).\]

Posons \(I_n=\int_0^n (1-t/n)^n\log(t)dt\), il nous reste à prouver que \(\lim_n I_n=-\gamma\). Pour cela, on fait dans \(I_n\) le changement de variables \(u=1-t/n\) : \[\begin{aligned}I_n&=\int_0^n \left(1-\dfrac{t}{n}\right)^n\log(t)dt =\int_0^1\, u^n\log(n(1-u))ndt\\ &=\int_0^1\, u^n\log(n)ndt+\int_0^1\, u^n\log(1-u)ndt\\ &=\dfrac{n\log(n)}{n+1}-n\int_0^1\, u^n\sum_{k=0}^\infty \dfrac{u^k}{k} dt\\ &=\dfrac{n\log(n)}{n+1}-\lim_{N\to\infty} \sum_{k=0}^N n\int_0^1\,\dfrac{u^{n+k}}{k}du\\ &=\dfrac{n\log(n)}{n+1}-\lim_{N\to\infty} \sum_{k=0}^N\dfrac{n}{k(n+k+1)}=\dfrac{n\log(n)}{n+1}-\lim_{N\to\infty}J_N\\ \end{aligned}\]\[J_N=\sum_{k=0}^N\dfrac{n}{k(n+k+1)} =\dfrac{n}{n+1}\sum_{k=0}^N\left(\dfrac{1}{k}-\dfrac{1}{n+k+1}\right).\] Pour \(N>n\) on peut écrire \[\begin{aligned}J_N&=\dfrac{n}{n+1}\left(\dfrac{1}{1}+\dots+\dfrac{1}{n-1}+\dfrac{1}{n}+\dfrac{1}{n+1}+\dots+\dfrac{1}{N} -\dfrac{1}{n+1}-\dots-\dfrac{1}{N}-\dfrac{1}{N+1}-\dots-\dfrac{1}{N+n+1}\right)\\ &=\dfrac{n}{n+1}\left[1+\dots+\dfrac{1}{n}- \underset{n+1\ \text{termes}}{\underbrace{\left(\dfrac{1}{N+1}+\dots+\dfrac{1}{N+n+1}\right)}}\right] \end{aligned}\] le groupe termes sur l’accolade est constitué de \(n+1\) termes, indépendant de \(N\) : il tendra donc vers \(0\) avec \(N\), soit \[\lim_{N\to\infty}J_N=\dfrac{n}{n+1}\left(1+\dots+\dfrac{1}{n}\right)\] puis \[I_n=\dfrac{n\log(n)}{n+1}-\lim_{N\to\infty}J_N=\dfrac{n\log(n)}{n+1}- \dfrac{n}{n+1}\left(1+\dots+\dfrac{1}{n}\right).\] et finalement \[\begin{aligned}\Gamma'(1)&=\lim_{n\to\infty} I_n =\lim_{n\to\infty}\dfrac{n\log(n)}{n+1}- \dfrac{n}{n+1}\left(1+\dots+\dfrac{1}{n}\right)\\ &=\lim_{n\to\infty}\dfrac{n}{n+1}\left(\log(n)-\left(1+\dots+\dfrac{1}{n}\right)\right)=-\gamma.\end{aligned}\]


  1. 1  Mais attention, vérifier les hypothèses de dérivation sous l’intégrale n’est pas compétement immédiat et doit être parfaitement maitrisé...!

\(\int_\mathbb R \vert f(t)\vert dt\leq \sqrt{8}\left( \int_\mathbb R \vert t f(t)\vert^2 dt\right)^{\frac{1}{4}}\left( \int_\mathbb R \vert f(t)\vert^2 dt\right)^{\frac{1}{4}}.\) *

9 novembre 2022 16:18 — Par Patrice Lassère

Soit \(f\in L^1(\mathbb R)\), montrer que \[\int_\mathbb R \vert f(t)\vert dt\leq \sqrt{8}\left( \int_\mathbb R \vert t f(t)\vert^2 dt\right)^{\frac{1}{4}}\left( \int_\mathbb R \vert f(t)\vert^2 dt\right)^{\frac{1}{4}}.\]



[ID: 2821] [Date de publication: 9 novembre 2022 16:18] [Catégorie(s): Intégration ] [ Nombre commentaires: 0] [nombre d'éditeurs: 1 ] [Editeur(s): Emmanuel Vieillard-Baron ] [nombre d'auteurs: 1 ] [Auteur(s): Patrice Lassère ]
Accordéon
Titre
Solution
Texte

\(\int_\mathbb R \vert f(t)\vert dt\leq \sqrt{8}\left( \int_\mathbb R \vert t f(t)\vert^2 dt\right)^{\frac{1}{4}}\left( \int_\mathbb R \vert f(t)\vert^2 dt\right)^{\frac{1}{4}}.\)
Par Patrice Lassère le 9 novembre 2022 16:18

Il n’y a bien sûr pas de raisons que \(f\in L^1(\mathbb R)\) assure la convergence des deux intégrales dans le terme de droite de l’inégalité. Dans le cas où au moins l’une de ces deux intégrales diverge on convient que l’inégalité s’écrit \(\int_\mathbb R \vert f(t)\vert dt\leq +\infty\) et il n’y a rien à démontrer. On peut donc dorénavant, supposer que ces trois intégrales convergent.

Soit \(x\in\mathbb R_+^\star\), pour faire apparaitre un terme du type \(tf(t)\), il convient d’isoler l’origine : \[\begin{aligned} \int_\mathbb R\vert f(t)\vert dt&=\int_{[-x,x]} \vert f(t)\vert dt+\int_{\mathbb R\setminus [-x,x]}\dfrac{1}{\vert t\vert}\vert tf(t)\vert dt\\ &\leq \sqrt{2x}\sqrt{\int_{[-x,x]} \vert f(t)\vert^2 dt}+\sqrt{\dfrac{2}{x}}\sqrt{\int_{\mathbb R\setminus [-x,x]}\vert tf(t)\vert^2 dt}\\ &\leq \sqrt{2Ax}+ \sqrt{\dfrac{2B}{x}}:=\varphi(x),\quad\forall\,x\in\mathbb R_+^\star \end{aligned}\] avec \(A= \int_\mathbb R \vert f(t)\vert^2 dt,\ B=\int_\mathbb R \vert t f(t)\vert^2 dt\), et où on a appliqué l’inégalité de Cauchy-Schwarz dans les deux intégrales pour obtenir la seconde inégalité. Ainsi \[\int_\mathbb R\vert f(t)\vert dt\leq \inf_{x\in\mathbb R_+^\star}\varphi(x)\] et déterminer cet infimum ne pose aucun problème puisque \(\varphi\) tends vers \(+\infty\) lorsque \(x\) tends vers \(0\) et \(+\infty\) et que sa dérivée ne s’annule qu’au point \(x=\sqrt{B/A}\) soit \(\varphi(\sqrt{B/A})=\inf_{x\in\mathbb R_+^\star}\varphi(x)=\sqrt{8}A^{1/4}B^{1/4}\), soit l’inégalité demandée.


Encore une petite inégalité *

9 novembre 2022 16:18 — Par Patrice Lassère

Soit \(f\in\mathscr C([0,1],[m,M])\) vérifiant \(\ \displaystyle\int_a^b f(t)dt=0\). Montrer que \[\int_a^b f^2(t)dt\leq -mM(b-a).\]



[ID: 2823] [Date de publication: 9 novembre 2022 16:18] [Catégorie(s): Intégration ] [ Nombre commentaires: 0] [nombre d'éditeurs: 1 ] [Editeur(s): Emmanuel Vieillard-Baron ] [nombre d'auteurs: 1 ] [Auteur(s): Patrice Lassère ]
Accordéon
Titre
Solution
Texte

Encore une petite inégalité
Par Patrice Lassère le 9 novembre 2022 16:18

Vu les hypothèses \[\int_a^b (f(t)-m)(M-f(t))dt\geq 0.\] Soit \[(M+m)\int_a^b f(t)dt-\int_a^bf^2(t)dt-mM(b-a)=-\int_a^bf^2(t)dt-mM(b-a)\geq 0,\] i.e. \[\int_a^bf^2(t)dt\leq -mM(b-a).\] Q.E.D.


Nature d’une intégrale impropre *

9 novembre 2022 16:18 — Par Patrice Lassère

Pour quels couples \((a,b)\in\mathbb R_+^\star\times\mathbb R_+^\star\) l’intégrale impropre \[\int_b^\infty\,\left( \sqrt{\sqrt{x+a}-\sqrt{x}}-\sqrt{\sqrt{x}-\sqrt{x-b}}\right)dx\] converge ?



[ID: 2825] [Date de publication: 9 novembre 2022 16:18] [Catégorie(s): Intégration ] [ Nombre commentaires: 0] [nombre d'éditeurs: 1 ] [Editeur(s): Emmanuel Vieillard-Baron ] [nombre d'auteurs: 1 ] [Auteur(s): Patrice Lassère ]
Accordéon
Titre
Solution
Texte

Nature d’une intégrale impropre
Par Patrice Lassère le 9 novembre 2022 16:18

L’intégrale est visiblement impropre uniquement en \(+\infty\), en utilisant répétitivement le développement limité \(\sqrt{1+t}=1+t/2+O(t^2)\) on a \[\begin{aligned} \sqrt{\sqrt{x+a}-\sqrt{x}}&=x^{1/4}\sqrt{\sqrt{1+\dfrac{a}{x}}-1}\\ &=x^{1/4}\sqrt{\dfrac{a}{2x}+O(x^{-2})}\\ &=\sqrt{\dfrac{a}{2}}x^{-1/4}(1+O(x^{-1})) \end{aligned}\] et \[\begin{aligned} \sqrt{\sqrt{x}-\sqrt{x-b}}&=x^{1/4}\sqrt{1-\sqrt{1-\dfrac{b}{x}}}\\ &=x^{1/4}\sqrt{\dfrac{b}{2x}+O(x^{-2})}\\ &=\sqrt{\dfrac{b}{2}}x^{-1/4}(1+O(x^{-1})). \end{aligned}\] Donc au voisinage de \(+\infty\) l’intégrande est équivalent à la fonction de signe constant (toujours au voisinage de \(+\infty\)) \[\left( \sqrt{\dfrac{a}{2}}-\sqrt{\dfrac{b}{2}}\right) x^{-1/4}+O(x^{-5/4}),\] et comme \(\int_b^\infty\,x^{-5/4}dx\) (car b>0) converge, notre intégrale sera convergente si et seulement si \(\int_b^\infty(\sqrt{a/2}-\sqrt{b/2})x^{-1/4}dx\) converge soit, si et seulement si \(a=b\) puisque \(\int_b^\infty\,x^{-1/4}dx\) diverge.


Une jolie intégrale.... *

9 novembre 2022 16:18 — Par Patrice Lassère

\(C(\alpha)\) désignant le coefficient de \(x^{2007}\) dans le développement en série entière de \((1+x)^\alpha\), calculer \[\int_0^1\,C(-t-1)\left( \dfrac{1}{t+1}+\dfrac{1}{t+2}+\dots+\dfrac{1}{t+2007}\right) dt.\]



[ID: 2827] [Date de publication: 9 novembre 2022 16:18] [Catégorie(s): Intégration ] [ Nombre commentaires: 0] [nombre d'éditeurs: 1 ] [Editeur(s): Emmanuel Vieillard-Baron ] [nombre d'auteurs: 1 ] [Auteur(s): Patrice Lassère ]
Accordéon
Titre
Solution
Texte

Une jolie intégrale....
Par Patrice Lassère le 9 novembre 2022 16:18

Nous connaissons le développement en série entière de \((1+x)^\alpha\) qui nous donne \[C(\alpha)=\dfrac{\alpha(\alpha-1)\dots(\alpha-2006)}{2007!}.\] Donc \(C(-t-1)=(t+1)(t+2)\dots(t+2007)/2007!\) et notre intégrande s’écrit \[\begin{aligned} C(-t-1)&\left( \dfrac{1}{t+1}+\dfrac{1}{t+2}+\dots+\dfrac{1}{t+2007}\right)\\ &=\dfrac{(t+1)(t+2)\dots(t+2007)}{2007} \left( \dfrac{1}{t+1}+\dfrac{1}{t+2}+\dots+\dfrac{1}{t+2007}\right)\\ &=\dfrac{(t+2)\dots(t+2007)+(t+1)(t+3)\dots(t+2007)+(t+1)(t+2)\dots(t+2006)}{2007!}\\ &=\dfrac{d}{dt}\left( \dfrac{(t+1)(t+2)\dots(t+2007)}{2007!}\right) . \end{aligned}\] Le calcul devient alors élémentaire \[\begin{aligned} \int_0^1\,C(-t-1)\left( \dfrac{1}{t+1}+\dfrac{1}{t+2}+\dots+\dfrac{1}{t+2007}\right) dt&=\int_0^1\,\dfrac{d}{dt}\left( \dfrac{(t+1)(t+2)\dots(t+2007)}{2007!}\right)dt\\ &=\left[ \dfrac{(t+1)(t+2)\dots(t+2007)}{2007!}\right]_0^1\\ &=\dfrac{2008!-2007!}{2007!}=2007. \end{aligned}\]

 Moralité :  Surtout ne pas se laisser impressionner !


Minore ! *

9 novembre 2022 16:18 — Par Patrice Lassère

Soit \(f\in\mathscr C^1(\mathbb R)\) , si \(f(1)=1\) et \[f'(x)=\dfrac{1}{x^2+f^2(x)},\quad\forall\,x\geq 1,\] montrer que \[\lim_{x\to+\infty}f(x)\leq 1+\dfrac{\pi}{4}.\]



[ID: 2829] [Date de publication: 9 novembre 2022 16:18] [Catégorie(s): Intégration ] [ Nombre commentaires: 0] [nombre d'éditeurs: 1 ] [Editeur(s): Emmanuel Vieillard-Baron ] [nombre d'auteurs: 1 ] [Auteur(s): Patrice Lassère ]
Accordéon
Titre
Solution
Texte

Minore !
Par Patrice Lassère le 9 novembre 2022 16:18

Une telle fonction sera visiblement strictement croissante, par conséquent \[f(x)>f(1)=1,\quad\forall\,x>1,\] puis \[f'(x)=\dfrac{1}{x^2+f^2(x)}<\dfrac{1}{1+x^2},\quad\forall\,x>1.\] Ainsi, pour \(x>1\) \[\begin{aligned}f(x)&=f(1)+\int_1^x\,f(t)dt\\ &<1+\int_1^x\dfrac{dt}{1+t^2}<1+\int_1^\infty\dfrac{dt}{1+t^2}=1+\dfrac{\pi}{4}. \end{aligned}\]


Autour des sommes de Riemann *

9 novembre 2022 16:18 — Par Patrice Lassère

[rms], 2008.

Soit \(f\in\mathscr C([0,1],\mathbb R^{\star +})\).

Pour tout \(n\in\mathbb N^\star\), établir l’existence de \((a_{n,0},\dots,a_{n,n})\in\mathbb R^{n+1}\) tel que \[0=a_{n,0}<\dots<a_{n,n}=1\ {\text{\ et\ }}\ \forall\,i\in\{0,\dots,n-1\}\ :\ \ \int_{a_{n,i}}^{a_{n,i+1}}\,f(t)dt=\dfrac{1}{n}\int_0^1f(t)dt.\]

Déterminer la limite, quand \(n\) tends vers \(+\infty\) de : \(\dfrac{a_{n,0}+\dots+a_{n,n}}{n+1}\).



[ID: 2831] [Date de publication: 9 novembre 2022 16:18] [Catégorie(s): Intégration ] [ Nombre commentaires: 0] [nombre d'éditeurs: 1 ] [Editeur(s): Emmanuel Vieillard-Baron ] [nombre d'auteurs: 1 ] [Auteur(s): Patrice Lassère ]
Accordéon
Titre
Solution
Texte

Autour des sommes de Riemann
Par Patrice Lassère le 9 novembre 2022 16:18

\(f\) étant continue sur \([0,1]\) à valeurs strictement positives l’application \(F\ :\ [0,1]\ni x\mapsto \int_0^x\, f(t)dt\) est continue, strictement croissante de \([0,1]\) sur \([0,\int_0^1\, f(t)dt]\) : par le théorème des valeurs intermédiaires, pour tout \(n\in\mathbb N\), il existe \(a_{n,0},\dots, a_{n,n}\) dans \([0,1]\) tels que \[F(a_{n,k})=\dfrac{k}{n}\int_0^{1}\,f(t)dt,\quad k=0,\dots,n.\] et ces réels sont uniques.

Vu ce qui précède, \(F^{-1}\in\mathscr C([0,\int_0^1\, f(t)dt], [0,1])\), la suite \((\frac{k}{n}\int_0^{1}\,f(t)dt)_{k=0}^n\) est une subdivision de l’intervalle \([0,\int_0^1\, f(t)dt]\), par conséquent \[\lim_{n\to\infty}\dfrac{a_{n,0}+\dots+a_{n,n}}{n+1}=\lim_{n\to\infty}\dfrac{1}{n+1}\sum_{k=0}^n\,F^{-1}\left( \dfrac{k}{n}\int_0^{1}\,f(t)dt\right) =\int_0^{\int_0^1f(u)du}\,F^{-1}(t)dt.\]


Autour d’Hölder *

9 novembre 2022 16:18 — Par Patrice Lassère

Soit \(f\in\mathscr C^0([0,1],\mathbb R)\), s’il existe \(a>0\) tel que \[0\leq f(x)\leq a^{2/3},\ \forall\,x\in[0,1],\quad {\text{et}}\quad \int_0^1\,f(x)dx=a,\] montrer à l’aide de l’inégalité de Hölder que \[\int_0^1\,\sqrt{f(x)}dx\geq a^{2/3}.\]



[ID: 2833] [Date de publication: 9 novembre 2022 16:18] [Catégorie(s): Intégration ] [ Nombre commentaires: 0] [nombre d'éditeurs: 1 ] [Editeur(s): Emmanuel Vieillard-Baron ] [nombre d'auteurs: 1 ] [Auteur(s): Patrice Lassère ]
Accordéon
Titre
Solution
Texte

Autour d’Hölder
Par Patrice Lassère le 9 novembre 2022 16:18

D’aprés l’inégalité d’Hölder1 nous avons pour \(p>1\) \[\begin{aligned}a=\int_0^1\, f(x)^{\frac{1}{2p}+1-\frac{1}{2p}}dx&\leq \left( \int_0^1\,\left[ f(x)^{\frac{1}{2p}}\right]^{p}dx\right)^{1/p} \left( \int_0^1\,\left[ f(x)^{1-\frac{1}{2p}}\right]^{\frac{p}{p-1}}dx\right)^{\frac{p-1}{p}}\\ &\leq \left( \int_0^1\,\sqrt{f(x)}dx\right)^{1/2p} \left( \int_0^1\,\left[ (a^{2/3})^{1-\frac{1}{2p}}\right]^{\frac{p}{p-1}}dx\right)^{\frac{p-1}{p}}\\ & =\left( \int_0^1\,\sqrt{f(x)}dx\right)^{1/2p} a^{\frac{2p-1}{3p}} \end{aligned}\] soit \[a^{\frac{p+1}{3}}\leq \int_0^1\,\sqrt{f(x)}dx,\quad\forall\,p>1.\] Comme \(p>1\) est arbitraire, le résultat suit en faisant tendre \(p\) vers \(1\).


  1. 1  Si \(f\in L^p(I),\ g\in L^q(I)\) alors \(fg\in L^1(I)\) et \(\int_I\vert f(x)g(x)\vert dx\leq (\int_I\vert f(x)\vert^p dx)^{1/p}(\int_I\vert f(x)\vert^q dx)^{1/q}\)\(p>1, q>1\) conjugués, et donner une référence....

Calcul de l’intégrale de Cauchy \(\int_0^{+\infty}\,{{\sin (t)}\over t}dt\) (8) *

9 novembre 2022 16:18 — Par Patrice Lassère

En intégrant \(f(x,t)=e^{-xy}sin(x)\) sur \([\epsilon, T]\times[0,+\infty[,\,0<\epsilon <T,\) calculer \(\displaystyle \int_0^{+\infty}\,{{\sin (t)}\over t}dt.\)



[ID: 2835] [Date de publication: 9 novembre 2022 16:18] [Catégorie(s): Intégration ] [ Nombre commentaires: 0] [nombre d'éditeurs: 1 ] [Editeur(s): Emmanuel Vieillard-Baron ] [nombre d'auteurs: 1 ] [Auteur(s): Patrice Lassère ]
Accordéon
Titre
Solution
Texte

Calcul de l’intégrale de Cauchy \(\int_0^{+\infty}\,{{\sin (t)}\over t}dt\) (8)
Par Patrice Lassère le 9 novembre 2022 16:18

Soient \(0<\varepsilon<T\), nous avons \[\begin{aligned}\int_\varepsilon^T\,\dfrac{\sin(x)}{x}dx&=\int_\varepsilon^T\,\sin(x)\left( \int_0^\infty\,e^{-xy}dy\right) dx\\ &=\int_0^\infty\,\left( \int_\varepsilon^T\,\sin(x)e^{-xy}dx\right) dy\\ &=\int_0^\infty\,\dfrac{e^{-y\varepsilon}(\cos\varepsilon+y\sin\varepsilon)-e^{-yT}(\cos T+y\sin T)}{y^2+1}dy\\ &=\int_0^\infty\,g_{\varepsilon,T}(y)dy \end{aligned}\] l’application ci-dessus du théorème de Fubini est justifiée par \(\vert f(x,y)\vert\leq e^{-xy}\) et \[\int_\varepsilon^T\,\left( \int_0^\infty\,e^{-xy}dy\right) dx= \int_\varepsilon^T\,\left[ -\dfrac{e^{xy}}{x}\right]_0^\infty dx=\int_\varepsilon^T\,\dfrac{dx}{x}=\log\dfrac{T}{\varepsilon}<\infty.\] pour tous \(0<\varepsilon<T\).

Maintenant, observons que pour \(0<\varepsilon\leq y\) \[\vert e^{-y\varepsilon}(\cos\varepsilon+y\sin\varepsilon)\vert\leq 1+y\varepsilon e^{-y\varepsilon}\leq 1+e^{-1},\] de même, pour \(T\geq 1\) \[\vert e^{-yT}(\cos T+y\sin T)\vert\leq e^{-yT}(1+y)\leq e^{-y}(1+y).\] Ainsi pour \(0<\varepsilon\leq y\leq T\) et \(T\leq 1\) \[\vert g_{\varepsilon,T}(y)\vert\leq \dfrac{\max\{(1+e^{-1}, e^{-y}(1+y)\}}{y^2+1}\in L^1(\mathbb R_+).\] Il est donc légitime d’invoquer le théorème de la convergence dominée pour écrire \[\lim_{\varepsilon\to 0_+}\lim_{T\to +\infty}\int_0^\infty\,g_{\varepsilon,T}(y)dy=\int_0^\infty\,\dfrac{dy}{y^2+1}=\dfrac{\pi}{2},\] d’autre part, comme \[\int_\varepsilon^T\,\dfrac{\sin(x)}{x}dx=\int_0^\infty\,g_{\varepsilon,T}(y)dy\] nous avons finalement \[\int_0^\infty\,\dfrac{\sin(x)}{x}dx=\lim_{\varepsilon\to 0_+}\lim_{T\to +\infty}\int_0^\infty\,g_{\varepsilon,T}(y)dy=\dfrac{\pi}{2}.\]


;
Success message!